Peds Exam 2 NCLEX

Lakukan tugas rumah & ujian kamu dengan baik sekarang menggunakan Quizwiz!

The nurse is admitting a child with a a dx. of mumps. What should the nurse do to prevent transmission?

*Airborne droplet and contact precautions *It is transmitted by droplet by infected person or direct contact * Negative pressure room; at least 12 exchanges per hr. *N95 respirator mask, gown, gloves

Patho for leukemia

- A group of malignant diseases of the bone marrow and lymphatic system - WBC overproduction occurs, although the count is actually low -o Immature cells do not deliberately attack and destroy normal cells but are in competition for metabolic elements

Manifestations for hemophilia

- History of bleeding, nosebleeds, and bruising - Hemarthrosis into the joint cavities of the knees, ankles, elbows -Hematoma with pain, swelling, and limited motion -Spontaneous hematuria

Diagnosis for Hodgkins and Non-hodgkins lymphoma

-Complete blood count (CBC) -Erythrocyte sedimentation rate (ESR) -Blood chemistry evaluation -Computed tomography (CT) chest x-ray -bone scan

What are s/s of Hirschsprung's disease?

1) Constipation 2) Abdominal distention 3) Ribbon-like stools with a foul smell

What are important nursing interventions for pyloric stenosis?

1) Hydration 2) I + O 3) Daily weights 4) Monitor urine specific gravity 5) Surgery possibly to open sphincter

What are treatments of intussusception?

1) Monitor stools - if returns to normal, then it is fixed 2) Barium enema to push out telescoped area 3) Surgery Keep in hospital for three days to ensure it does not come back

How do we treat celiac disease?

1) No food with gluten (vegetable proteins) 2) No BROW: -BARLEY -RYE -OATS -WHEAT 3) Eat RCS: -RICE -CORN -SOY

What are s/s of pyloric stenosis?

1) Projectile vomiting 2) Olive shaped mass in the epigastric region, near the umbilicus 3) Extreme hunger

How do we treat Hirschsprung's disease?

1) Remove the portion of the bowel that is diseased 2) May require two surgeries to give intestines time to heal

What are s/s of intussusception?

1) Sudden onset 2) Cramping 3) Intermittent abdominal pain 4) Inconsolability 5) Drawing up knees 6) *Currant jelly stools*

Dont Don't want to give with dairy because it will bind to calcium - give __ hour before or __ hours after milk consumption

1, 2

A nurse is caring for a child with von Willebrand disease. The nurse is aware that which of the following is a (are) clinical manifestation(s) of von Willebrand disease? Select all that apply. 1. Bleeding of the mucous membranes. 2. The child bruises easily. 3. Excessive menstruation. 4. The child has frequent nosebleeds. 5. Elevated creatinine levels. 6. The child has a factor IX deficiency.

1, 2, 3, 4. 1. Von Willebrand disease is a hereditary bleeding disorder characterized by deficiency of or defect in a protein. The disorder causes adherence of platelets to damaged endothelium and a mild deficiency of factor VIII. One of the manifestations of this disease is bleeding of the mucous membranes. 2. Bruising is a common manifestation of this disease. 3. Excessive menstruation may be a manifestation of this disease. 4. Frequent nosebleeds is a common manifestation of this disease. 5. There is no increase in creatinine in this disease. 6. Von Willebrand disease is a mild factor VIII deficiency, not a factor IX deficiency. TEST-TAKING HINT: Focus on the diagnosis. Von Willebrand disease is a minor deficiency of factor VIII, so the clinical manifestations will be less severe.

The nurse is caring for a child who is receiving a transfusion of packed red blood cells. The nurse is aware that if the child had a hemolytic reaction to the blood, the signs and symptoms would include which of the following? Select all that apply. 1. Fever. 2. Rash. 3. Oliguria. 4. Hypotension. 5. Chills.

1, 3, 4 1. Hemolytic reactions include fever, pain at insertion site, hypotension, renal failure, tachycardia, oliguria, and shock. 2. Febrile reactions are fever and chills. Allergic reactions include hives, itching and respiratory distress. 3. Hemolytic reactions include fever, pain at insertion site, hypotension, renal failure, tachycardia, oliguria, and shock. 4. Hemolytic reactions include fever, pain at insertion site, hypotension, renal failure, tachycardia, oliguria, and shock. 5. Febrile reactions are fever and rash. Allergic reactions include rash, hives, and respiratory distress. TEST-TAKING HINT: Review the signs and symptoms of hemolytic reaction, febrile reaction, and allergic reaction. Understanding the causes of the reactions will help identify the symptoms.

Which of the following factors need(s) to be included in a teaching plan for a child with sickle cell anemia? Select all that apply. 1. The child needs to be taken to a physician when sick. 2. The parent should make sure the child sleeps in an air-conditioned room. 3. Emotional stress should be avoided. 4. It is important to keep the child well hydrated. 5. It is important to make sure the child gets adequate nutrition.

1, 3, 4, 5. 1. The goal of therapy with children is to prevent the sickling process. 2. A cold environment causes vasoconstriction, which needs to be prevented to get good tissue perfusion. 3. Seek medical attention for illness to prevent the child from going into a crisis. 4. The child needs good hydration and nutrition to maintain good health. 5. The child needs good hydration and nutrition to maintain good health. TEST-TAKING HINT: Focus on how to prevent a sickle cell crisis.

The nurse is performing an assessment on a 10-year-old child suspected to have Hodgkin's disease. Which assessment findings are specifically characteristic of this disease? Select all that apply. 1. Abdominal pain 2. Fever and malaise 3. Anorexia and weight loss 4. Painful, enlarged inguinal lymph nodes 5. Painless, firm, and movable adenopathy in the cervical area

1. Abdominal pain 5. Painless, firm, and movable adenopathy in the cervical area Hodgkin's disease (a type of lymphoma) is a malignancy of the lymph nodes. Specific clinical manifestations associated with Hodgkin's disease include painless, firm, and movable adenopathy in the cervical and supraclavicular areas and abdominal pain as a result of enlarged retroperitoneal nodes. Hepatosplenomegaly also is noted. Although fever, malaise, anorexia, and weight loss are associated with Hodgkin's disease, these manifestations are seen in many disorders.

When caring for a child with submersion injury, the nurse should do which of the following? (Select all that apply.) 1. Administer oxygen as ordered. 2. Administer furosemide as ordered. 3. Maintain mechanical ventilation. 4. Monitor child for increased intracranial pressure. 5. Monitor electrolyte status.

1. Administer oxygen as ordered. 2. Administer furosemide as ordered. 3. Maintain mechanical ventilation. 4. Monitor child for increased intracranial pressure. 5. Monitor electrolyte status. Children with submersion injury will require nursing care specific to their symptomatology.

Which of the following should be avoided if the child has hypospadias? 1. Circumcision 2. Catheterization 3. Surgery 4. Intravenous pyelography (IVP)

1. Circumcision Hypospadias refers to a condition in which the urethral opening is located below the glans penis or anywhere along the ventral surface (underside) of the penile shaft. The ventral foreskin is lacking, and the distal portion gives an appearance of a hood. Early recognition is important so that circumcision is avoided; the foreskin is used for the surgical repair. Catheterization may be used to ensure urinary elimination. Surgery is the procedure of choice to improve the child's ability to stand when urinating, improve the appearance of the penis, and preserve sexual adequacy. IVP is contraindicated if the child has an allergy to iodine or shellfish.

Which of the following is the most useful tool in diagnosing seizure disorder? 1. EEG 2. Lumbar puncture 3. Brain scan 4. Skull radiographs

1. EEG The EEG detects abnormal electrical activity in the brain. The pattern of various spikes can aid in the diagnosis of specific seizure disorders. Lumbar puncture confirms problems related to cerebrospinal fluid infection or trauma. Brain scans confirm space-occupying lesions. Skull radiographs can detect fractures and structural abnormalities.

When caring for a child awaiting surgery for a Wilms' tumor, which of the following nursing actions would be most important? 1. Handling the child with care, particularly during bathing 2. Placing the child on low blood count precautions and isolation 3. Monitoring bowel sounds for vincristine-induced ileus 4. Placing the child in high Fowler's position to facilitate breathing

1. Handling the child with care, particularly during bathing Handling the child carefully is essential to prevent rupture of an encapsulated tumor. The child usually does not undergo myelosuppression before surgery. In fact, the child may be suffering from polycythemia due to increased production of erythropoietin. Vincristine usually is administered postoperatively. Respiratory difficulty is not a common problem with Wilms' tumor.

A neonate's failure to pass meconium within the first 24 hours after birth may indicate which of the following? 1. Hirschsprung's disease 2. Celiac disease 3. Intussusception 4. Abdominal wall defect

1. Hirschsprung's disease Failure to pass meconium within the first 24 hours after birth may be an indication of Hirschsprung's disease, a congenital anomaly resulting in mechanical obstruction due to inadequate motility in an intestinal segment. Failure to pass meconium is not associated with celiac disease, intussusception, or abdominal wall defect.

The nurse analyzes the laboratory values of a child with leukemia who is receiving chemotherapy. The nurse notes that the platelet count is 19,500 mm3 (19.5 × 109/L). On the basis of this laboratory result, which intervention should the nurse include in the plan of care? 1. Initiate bleeding precautions. 2. Monitor closely for signs of infection. 3. Monitor the temperature every 4 hours. 4. Initiate protective isolation precautions.

1. Initiate bleeding precautions. Leukemia is a malignant increase in the number of leukocytes, usually at an immature stage, in the bone marrow. It affects the bone marrow, causing anemia from decreased erythrocytes, infection from neutropenia, and bleeding from decreased platelet production (thrombocytopenia). If a child is severely thrombocytopenic and has a platelet count less than 20,000 mm3 (20.0 × 109/L), bleeding precautions need to be initiated because of the increased risk of bleeding or hemorrhage. Precautions include limiting activity that could result in head injury, using soft toothbrushes, checking urine and stools for blood, and administering stool softeners to prevent straining with constipation. In addition, suppositories, enemas, and rectal temperatures are avoided. Options 2, 3, and 4 are related to the prevention of infection rather than bleeding.

3 types of non-hodgkin's lymphoma?

1. Mature B-cell identified as Burkitt's lymphoma 2. Lymphoblastic lymphoma of precursor T cell and precursor B cell 3.Anaplastic large-cell

A 2-year-old boy with a diagnosis of hemophilia is admitted to the hospital with bleeding into the joint of the right knee. Which intervention should the nurse plan to implement with this child? 1. Measure the injured knee joint every shift. 2. Take the temperature by rectal method only. 3. Administer acetylsalicylic acid for pain control. 4. Immobilize the joint and apply moist heat to the joint.

1. Measure the injured knee joint every shift. Interventions for bleeding into the joint include measuring the injured joint to assess for progression of the bleeding. This provides objective rather than subjective data, which are needed to determine if the bleeding is increasing. Rectal temperatures can cause tissue trauma, causing further bleeding. The application of heat and the administration of acetylsalicylic acid will increase bleeding. Cognitive Ability: Applying Client Needs: Physiological Integrity Integrated Process: Nursing Process: Planning Content Area: Pediatrics: Hematological Strategy(ies): Closed-ended Word, Subject Priority Concepts: Clotting, Safety

The nurse notes documentation that a child is exhibiting an inability to flex the leg when the thigh is flexed anteriorly at the hip. Which condition does the nurse suspect? 1. Meningitis 2. Spinal cord injury 3. Intracranial bleeding 4. Decreased cerebral blood flow

1. Meningitis Meningitis is an infectious process of the central nervous system caused by bacteria and viruses. The inability to extend the leg when the thigh is flexed anteriorly at the hip is a positive Kernig's sign, noted in meningitis. Kernig's sign is not seen specifically with spinal cord injury, intracranial bleeding, or decreased cerebral blood flow.

28.The nurse assesses the following blood gas results on an infant in the emergency department. Which of the following conclusions is consistent with the data? ● Po2: 90 mm Hg ● Pco2: 34 mm Hg ● HCO3: 16 mEq/L ● Base excess: − 4 ● pH: 7.28 1. Metabolic acidosis 2. Metabolic alkalosis 3. Respiratory acidosis 4. Respiratory alkalosis

1. Metabolic acidosis 1. Check the pH: 7.28 is an acidic pH. 2. Check Pco2: low 3. Using ROME, the pH and the Pco2 are both low, i.e., they are altered in the same direction. The cause of the altered blood gases, therefore, is metabolic. 4. Check HCO3: low = cause of the problem. 5. Low Pco2 = compensatory response. 6. The child is in metabolic acidosis. A possible medical diagnosis for the metabolic acidosis is diarrhea.

What should the nurse do first after noting that a child with Hirschsprung's disease has a fever and watery explosive diarrhea? 1. Notify the health care provider immediately. 2. Administer an antidiarrheal. 3. Monitor the child every 30 minutes. 4. Nothing. (This is characteristic of Hirschsprung's disease.)

1. Notify the health care provider immediately. For the child with Hirschsprung's disease, fever and explosive diarrhea indicate enterocolitis, a life-threatening situation. Therefore, the health care provider should be notified immediately. Generally, because of the intestinal obstruction and inadequate propulsive intestinal movement, antidiarrheals are not used to treat Hirschsprung's disease. The child is acutely ill and requires intervention, with monitoring more frequently than every 30 minutes. Hirschsprung's disease typically presents with chronic constipation.

The clinic nurse is assessing a child for dehydration. The nurse determines that the child is moderately dehydrated if which finding is noted on assessment? 1. Oliguria 2. Flat fontanels 3. Pale skin color 4. Moist mucous membranes

1. Oliguria In moderate dehydration, the fontanels would be slightly sunken, the mucous membranes would be dry, and the skin color would be dusky. Also, oliguria would be present. Cognitive Ability: Analyzing Client Needs: Physiological Integrity Integrated Process: Nursing Process: Assessment Content Area: Pediatrics: Metabolic/Endocrine Strategy(ies): Subject Priority Concepts: Clinical Judgment, Fluid and Electrolyte Balance

The nurse is performing an assessment on a child admitted to the hospital with a probable diagnosis of nephrotic syndrome. Which assessment findings should the nurse expect to observe? Select all that apply. 1. Pallor 2. Edema 3. Anorexia 4. Proteinuria 5. Weight loss 6. Decreased serum lipids

1. Pallor 2. Edema 3. Anorexia 4. Proteinuria Nephrotic syndrome is a kidney disorder characterized by massive proteinuria, hypoalbuminemia, edema, elevated serum lipids, anorexia, and pallor. The child gains weight.

The mother of a 4-year-old child tells the pediatric nurse that the child's abdomen seems to be swollen. During further assessment, the mother tells the nurse that the child is eating well and that the activity level of the child is unchanged. The nurse, suspecting the possibility of Wilms' tumor, should avoid which during the physical assessment? 1. Palpating the abdomen for a mass 2. Assessing the urine for the presence of hematuria 3. Monitoring the temperature for the presence of fever 4. Monitoring the blood pressure for the presence of hypertension

1. Palpating the abdomen for a mass Wilms' tumor is the most common intra-abdominal and kidney tumor of childhood. If Wilms' tumor is suspected, the tumor mass should not be palpated by the nurse. Excessive manipulation can cause seeding of the tumor and spread of the cancerous cells. Hematuria, fever, and hypertension are clinical manifestations associated with Wilms' tumor.

Chronic myeloblastic (CML)

1. Patho = Philadelphia chromosome abnormality, radiation exposure 2. Fever without infection, bone pain, swollen spleen with pain, night sweats, rash 3. Treat with Imantinib (gleevec) oral pill, bone marrow transplant

Acute myeloblastic (AML) → more severe

1. Patho = certain chemicals, chemotherapy drugs, radiation 2. Bleeding from nose or gums, fever, SOB, skin rash 3. Treat with antibiotics, bone marrow transplant, blood and platelet transfusion

Acute lymphocytic (ALL)

1. Patho = chromosome problems, radiation or chemo 2. Bone and joint pain, bruising, bleeding, fatigue, swollen glands, weight loss 3. Treat with chemo, blood transfusion, antibiotics, bone marrow transplant

o Chronic Lymphocytic (CLL)

1. Patho = unknown, common in adults 70 and older 2. Bruising, enlarged lymph nodes, fever, infections that recur, weight loss, night sweats 3. No treatment in early stage → later stage chemo, radiation, and bone marrow transplant

When planning a client education program for sickle cell disease, the nurse should include which of the following topics? 1. Proper hand washing and infection avoidance 2. A high-iron, high-protein diet 3. Fluid restriction to 1 qt (1 l)/day 4. Aerobic exercises to increase oxygenation

1. Proper hand washing and infection avoidance Prevention of infection is an important measure in the prevention of sickle cell crisis. A high-iron, high-protein diet would have no effect on the disease or prevention of a crisis. Proper hydration should be encouraged to prevent crisis secondary to dehydration. Strenuous exercise and activity should be avoided to reduce the risk of increased tissue ischemia.

35.The nurse, who is assessing the blood gas results of a young child in the emergency department, notes that the Pco2 is elevated and that the pH is low. The nurse will check to see if the child's body has attempted to compensate for the disturbance by doing which of the following? 1. Raising the serum bicarbonate levels 2. Raising the serum oxygen levels 3. Raising the serum carbonic acid levels 4. Raising the serum potassium levels

1. Raising the serum bicarbonate levels To compensate for respiratory acidosis, the body should try to compensate by raising the bicarbonate levels.

The nurse is reviewing a health care provider's prescriptions for a child with sickle cell anemia who was admitted to the hospital for the treatment of vaso-occlusive crisis. Which prescriptions documented in the child's record should the nurse question? Select all that apply. 1. Restrict fluid intake. 2. Position for comfort. 3. Avoid strain on painful joints. 4. Apply nasal oxygen at 2 L/minute. 5. Provide a high-calorie, high-protein diet. 6. Give meperidine, 25 mg intravenously, every 4 hours for pain.

1. Restrict fluid intake. 6. Give meperidine, 25 mg intravenously, every 4 hours for pain. Sickle cell anemia is one of a group of diseases termed hemoglobinopathies, in which hemoglobin A is partly or completely replaced by abnormal sickle hemoglobin S. It is caused by the inheritance of a gene for a structurally abnormal portion of the hemoglobin chain. Hemoglobin S is sensitive to changes in the oxygen content of the red blood cell; insufficient oxygen causes the cells to assume a sickle shape, and the cells become rigid and clumped together, obstructing capillary blood flow. Oral and intravenous fluids are an important part of treatment. Meperidine is not recommended for a child with sickle cell disease because of the risk for normeperidine-induced seizures. Normeperidine, a metabolite of meperidine, is a central nervous system stimulant that produces anxiety, tremors, myoclonus, and generalized seizures when it accumulates with repetitive dosing. The nurse would question the prescription for restricted fluids and meperidine for pain control. Positioning for comfort, avoiding strain on painful joints, oxygen, and a high-calorie and high-protein diet are also important parts of the treatment plan.

The nurse is planning care for a child with hemolytic-uremic syndrome who has been anuric and will be receiving peritoneal dialysis treatment. The nurse should plan to implement which measure? 1. Restrict fluids as prescribed. 2. Care for the arteriovenous fistula. 3. Encourage foods high in potassium. 4. Administer analgesics as prescribed.

1. Restrict fluids as prescribed. Hemolytic-uremic syndrome is thought to be associated with bacterial toxins, chemicals, and viruses that result in acute kidney injury in children. Clinical manifestations of the disease include acquired hemolytic anemia, thrombocytopenia, renal injury, and central nervous system symptoms. A child with hemolytic-uremic syndrome undergoing peritoneal dialysis because of anuria would be on fluid restriction. Pain is not associated with hemolytic-uremic syndrome, and potassium would be restricted, not encouraged, if the child is anuric. Peritoneal dialysis does not require an arteriovenous fistula (only hemodialysis).

While assessing a neonate with cleft lip (CL), the nurse would be alert that which of the following will most likely be compromised? 1. Sucking ability 2. Respiratory status 3. Locomotion 4. GI function

1. Sucking ability Because of the defect, the child will be unable to form the mouth adequately around the nipple, thereby requiring special devices to allow for feeding and sucking gratification. Respiratory status may be compromised if the child is fed improperly or during postoperative period. Locomotion would be a problem for the older infant because of the use of restraints. GI functioning is not compromised in the child with a CL.

The nurse is creating a plan of care for a child who is at risk for seizures. Which interventions apply if the child has a seizure? Select all that apply. 1. Time the seizure. 2. Restrain the child. 3. Stay with the child. 4. Place the child in a prone position. 5. Move furniture away from the child. 6. Insert a padded tongue blade in the child's mouth.

1. Time the seizure. 3. Stay with the child. 5. Move furniture away from the child. A seizure is a disorder that occurs as a result of excessive and unorganized neuronal discharges in the brain that activate associated motor and sensory organs. During a seizure, the child is placed on his or her side in a lateral position. Positioning on the side prevents aspiration because saliva drains out the corner of the child's mouth. The child is not restrained because this could cause injury to the child. The nurse would loosen clothing around the child's neck and ensure a patent airway. Nothing is placed into the child's mouth during a seizure because this action may cause injury to the child's mouth, gums, or teeth. The nurse would stay with the child to reduce the risk of injury and allow for observation and timing of the seizure.

The development of Reye's syndrome has been associated with the use of aspirin and which of the following? 1. Varicella 2. Meningitis 3. Encephalitis 4. Strep throat

1. Varicella Reye's syndrome has been associated with the ingestion of aspirin in children with viral infections such as varicella. There is no association between meningitis or bacterial infections such as strep throat and the development of Reye's syndrome. Encephalitis is a component of Reye's syndrome.

The nurse is monitoring a 3-year-old child for signs and symptoms of increased intracranial pressure (ICP) after a craniotomy. The nurse plans to monitor for which early sign or symptom of increased ICP? 1. Vomiting 2. Bulging anterior fontanel 3. Increasing head circumference 4. Complaints of a frontal headache

1. Vomiting The brain, although well protected by the solid bony cranium, is highly susceptible to pressure that may accumulate within the enclosure. Volume and pressure must remain constant within the brain. A change in the size of the brain, such as occurs with edema or increased volume of intracranial blood or cerebrospinal fluid without a compensatory change, leads to an increase in ICP, which may be life-threatening. Vomiting, an early sign of increased ICP, can become excessive as pressure builds up and stimulates the medulla in the brainstem, which houses the vomiting center. Children with open fontanels (posterior fontanel closes at 2 to 3 months; anterior fontanel closes at 12 to 18 months) compensate for ICP changes by skull expansion and subsequent bulging fontanels. When the fontanels have closed, nausea, excessive vomiting, diplopia, and headaches become pronounced, with headaches becoming more prevalent in older children.

Which of the following parameters would the nurse monitor to evaluate the effectiveness of thickened feedings for an infant with gastroesophageal reflux (GER)? 1. Vomiting 2. Stools 3. Urine 4. Weight

1. Vomiting Thickened feedings are used with GER to stop the vomiting. Therefore, the nurse would monitor the child's vomiting to evaluate the effectiveness of using the thickened feedings. No relationship exists between feedings and characteristics of stools and urine. If feedings are ineffective, this should be noted before there is any change in the child's weight.

A kid with rubeola (measles) is being admitted to hosp. The nurse should plan for which precaution 1. enteric 2. airborne 3. protective 4. neutropenic

2 * rubeola is transmitted via airborne particles and dirsct contact with the infectious drops. People involved with the kid should wear a mask, private room, door remains closed,

Which of the following describe(s) ITP? Select all that apply. 1. ITP is a congenital hematological disorder. 2. ITP causes excessive destruction of platelets. 3. Children with ITP have normal bone marrow. 4. Platelets are small in ITP. 5. Purpura is observed in ITP

2, 3, 5. 1. ITP is an acquired hematological condition that is characterized by excessive destruction of platelets, purpura, and normal bone marrow along with increase in large, yellow platelets. 2. ITP is characterized by excessive destruction of platelets. 3. The bone marrow is normal in children with ITP. 4. Platelets are large, not small. 5. ITP is characterized by purpuras, which are areas of hemorrhage under the skin. TEST-TAKING HINT: Review the pathophysiology of ITP to determine the manifestations of the disease.

A nurse is assessing a kid who is scheduled to receive a live vaccine. What are the general contraindications associated with receiving a live virus vaccine? SELECT ALL 1. The kid has symptoms of a cold 2. The child has a previous anaphylactic reaction to the vaccine 3. mother reports the kid is having intermittent episodes of diarrhea 4. mother reports that the kid has had no appetite and has been fussy 5. The kid has a severely deficient immune system 6. Mother reports that the kid has recently been exposed to infectious disease

2, 5 The contraindications for for this are a previous anaphylactic episode, weakened immune systems, those with a severe sensitivity to gelatin, or pregnant women

The nurse reviews the record of a child who is suspected to have glomerulonephritis. Which statement by the child's parent should the nurse expect that is associated with this diagnosis? 1. "I'm so glad they didn't find any protein in his urine." 2. "I noticed his urine was the color of coca-cola lately." 3. "His health care provider said his kidneys are working well." 4. "The nurse who admitted my child said his blood pressure was low."

2. "I noticed his urine was the color of coca-cola lately." Glomerulonephritis refers to a group of kidney disorders characterized by inflammatory injury in the glomerulus. Gross hematuria, resulting in dark, smoky, cola-colored or brown-colored urine, is a classic symptom of glomerulonephritis. Blood urea nitrogen levels and serum creatinine levels may be elevated, indicating that kidney function is compromised. A mild to moderate elevation in protein in the urine is associated with glomerulonephritis. Hypertension is also common due to fluid volume overload secondary to the kidneys not working properly.

The nurse provides a teaching session to the nursing staff regarding osteosarcoma. Which statement by a member of the nursing staff indicates a need for information? 1. "The femur is the most common site of this sarcoma." 2. "The child does not experience pain at the primary tumor site." 3. "Limping, if a weight-bearing limb is affected, is a clinical manifestation." 4. "The symptoms of the disease in the early stage are almost always attributed to normal growing pains."

2. "The child does not experience pain at the primary tumor site." Osteosarcoma is the most common bone cancer in children. Cancer usually is found in the metaphysis of long bones, especially in the lower extremities, with most tumors occurring in the femur. Osteosarcoma is manifested clinically by progressive, insidious, and intermittent pain at the tumor site. By the time these children receive medical attention, they may be in considerable pain from the tumor. Options 1, 3, and 4 are accurate regarding osteosarcoma.

The nurse advises the parents of a 1½-year-old who is newly diagnosed with type 1 diabetes that the child's blood glucose level before dinner should be between 90 and 140 mg/dL. The mother states, "But that is much higher than I read on an Internet Web site." Which of the following responses by the nurse is appropriate? 1. "I am sorry, I was thinking of the level for after dinner. The correct before dinner level is 70 to 110 mg/dL." 2. "The level is higher than what you will usually see because young children's diets are not as predictable as the diets of older children and adults." 3. "The level before breakfast should be 70 to 100 mg/dL, but the before dinner level should be a higher level." 4. "You will find that your primary health-care provider will change the level at each visit. The goal starts at a high level and drops as your child responds to the insulin."

2. "The level is higher than what you will usually see because young children's diets are not as predictable as the diets of older children and adults." Toddlers often go through a stage when they are finicky eaters. They are, therefore, at high risk for becoming hypoglycemic. The higher preprandial blood glucose level helps to reduce the risk of developing low blood glucose levels.

An 11-year-old child is admitted to the hospital in vaso-occlusive sickle cell crisis. The nurse plans for which priority treatments in the care of the child? 1. Splenectomy, correction of acidosis 2. Adequate hydration, pain management 3. Frequent ambulation, oxygen administration 4. Passive range-of-motion exercises, adequate hydration

2. Adequate hydration, pain management During vaso-occlusive sickle cell crisis, the care focuses on adequate hydration and pain management. Adequate hydration with intravenous normal saline and oral fluids maintains blood flow and decreases the severity of the vaso-occlusive crisis. Analgesics for pain management are necessary during a vaso-occlusive crisis. Splenectomy would not be done with a vaso-occlusive crisis. Acidosis is not present. Oxygen can be administered to increase tissue perfusion but is not the priority treatment for a vaso-occlusive crisis. Passive range of motion is not recommended; bed rest is prescribed initially. Cognitive Ability: Analyzing Client Needs: Physiological Integrity Integrated Process: Nursing Process: Planning Content Area: Pediatrics: Hematological Strategy(ies): Strategic Words Priority Concepts: Clinical Judgment, Gas Exchange

The nurse is instructing the parents of a child with iron deficiency anemia regarding the administration of a liquid oral iron supplement. Which instruction should the nurse tell the parents? 1. Administer the iron at mealtimes. 2. Administer the iron through a straw. 3. Mix the iron with cereal to administer. 4. Add the iron to formula for easy administration.

2. Administer the iron through a straw. In iron deficiency anemia, iron stores are depleted, resulting in a decreased supply of iron for the manufacture of hemoglobin in red blood cells. An oral iron supplement should be administered through a straw or medicine dropper placed at the back of the mouth because the iron stains the teeth. The parents should be instructed to brush or wipe the child's teeth or have the child brush the teeth after administration. Iron is administered between meals because absorption is decreased if there is food in the stomach. Iron requires an acid environment to facilitate its absorption in the duodenum. Iron is not added to formula or mixed with cereal or other food items. Cognitive Ability: Applying Client Needs: Physiological Integrity Integrated Process: Teaching and Learning Content Area: Pediatrics: Hematological Strategy(ies): Comparable or Alike Options Priority Concepts: Client Education, Health Promotion

Which of the following would be the lowest priority when caring for a child with a seizure disorder? 1. Teaching the family about anticonvulsant therapy 2. Assessing for signs and symptoms of increased intracranial pressure (ICP) 3. Ensuring safety and protection from injury 4. Observing and recording all seizures

2. Assessing for signs and symptoms of increased intracranial pressure (ICP) Signs and symptoms of increased ICP are not associated with seizure activity and therefore would be the lowest priority. Improper administration of and incomplete compliance with anticonvulsant therapy can lead to status epilepticus; thus, education is a priority. Safety is always a priority in the care of a child with a seizure disorder because seizures may occur at any given time. Careful observation and documentation of seizures provide valuable information to aid prevention and treatment.

An adolescent with diabetes receives 30 units of Humulin N insulin at 7:00 a.m. The nurse would monitor for a hypoglycemic episode at what time? 1. At bedtime 2. Before supper 3. At midmorning 4. After breakfast

2. Before supper Humulin N insulin is an intermediate-acting insulin that peaks in approximately 6 to 12 hours. It would peak before supper if given at 7:00 a.m. Short-acting insulin would peak after breakfast or midmorning. Long-acting insulins would peak at bedtime. Cognitive Ability: Applying Client Needs: Physiological Integrity Integrated Process: Nursing Process: Assessment Content Area: Pediatrics: Metabolic/Endocrine Strategy(ies): Subject, Data in the Question Priority Concepts: Clinical Judgment, Glucose Regulation

The nursing student is assigned to care for a child with hemophilia. The nursing instructor reviews the plan of care with the student. Which intervention on the student written plan of care requires correction? 1. Measure circumference of injured joints. 2. Blood transfusion of packed red blood cells. 3. Monitor temperature with oral thermometers. 4. Intravenous administration of recombinant factor.

2. Blood transfusion of packed red blood cells. Blood product transfusion is not the treatment of choice over administering recombinant factors intravenously. Measuring circumference of injured joints is appropriate to assess for enlarging hematomas or bleeding under the skin. The nurse should avoid taking rectal temperatures to decrease the risk for injury. Cognitive Ability: Evaluating Client Needs: Physiological Integrity Integrated Process: Teaching and Learning Content Area: Pediatrics: Hematological Strategy(ies): Subject Priority Concepts: Clinical Judgment, Clotting

The nurse would prepare the parents of a child with suspected leukemia for which of the following tests that would confirm this diagnosis? 1. Lumbar puncture 2. Bone marrow aspiration 3. Complete blood count (CBC) with differential 4. Blood culture

2. Bone marrow aspiration A bone marrow aspiration is performed to confirm the diagnosis of leukemia through the examination of abnormal cells in the bone marrow. A lumbar puncture is performed to detect spread into the central nervous system, but it is not used to confirm the diagnosis. An abnormal CBC may suggest leukemia, but it is not used to confirm the diagnosis. A blood culture may be performed if infection is suspected.

A 4-year-old child is admitted to the hospital for abdominal pain. The mother reports that the child has been pale and excessively tired and is bruising easily. On physical examination, lymphadenopathy and hepatosplenomegaly are noted. Diagnostic studies are being performed because acute lymphocytic leukemia is suspected. The nurse determines that which laboratory result confirms the diagnosis? 1. Lumbar puncture showing no blast cells 2. Bone marrow biopsy showing blast cells 3. Platelet count of 350,000 mm3 (350 × 109/L) 4. White blood cell count 4500 mm3 (4.5 × 109/L)

2. Bone marrow biopsy showing blast cells Leukemia is a malignant increase in the number of leukocytes, usually at an immature stage, in the bone marrow. The confirmatory test for leukemia is microscopic examination of bone marrow obtained by bone marrow aspirate and biopsy, which is considered positive if blast cells are present. An altered platelet count occurs as a result of the disease, but also may occur as a result of chemotherapy and does not confirm the diagnosis. The white blood cell count may be normal, high, or low in leukemia. A lumbar puncture may be done to look for blast cells in the spinal fluid that indicate central nervous system disease.

The nurse is caring for an infant with a diagnosis of bladder exstrophy. To protect the exposed bladder tissue, the nurse should plan which intervention? 1. Cover the bladder with petroleum jelly gauze. 2. Cover the bladder with a nonadhering plastic wrap. 3. Apply sterile distilled water dressings over the bladder mucosa. 4. Keep the bladder tissue dry by covering it with dry sterile gauze.

2. Cover the bladder with a nonadhering plastic wrap. In bladder exstrophy, the bladder is exposed and external to the body. In this disorder, one must take care to protect the exposed bladder tissue from drying, while allowing the drainage of urine. This is accomplished best by covering the bladder with a nonadhering plastic wrap. The use of petroleum jelly gauze should be avoided because this type of dressing can dry out, adhere to the mucosa, and damage the delicate tissue when removed. Dry sterile dressings and dressings soaked in solutions (that can dry out) also damage the mucosa when removed.

The nurse performing an admission assessment on a 2-year-old child who has been diagnosed with nephrotic syndrome notes that which most common characteristic is associated with this syndrome? 1. Hypertension 2. Generalized edema 3. Increased urinary output 4. Frank, bright red blood in the urine

2. Generalized edema Nephrotic syndrome is defined as massive proteinuria, hypoalbuminemia, hyperlipemia, and edema. Other manifestations include weight gain; periorbital and facial edema that is most prominent in the morning; leg, ankle, labial, or scrotal edema; decreased urine output and urine that is dark and frothy; abdominal swelling; and blood pressure that is normal or slightly decreased.

Which of the following interventions would be included in the care plan of a child with sickle cell disease? 1. Administration of an anticoagulant to prevent sickling 2. Health teaching to help reduce sickling crises 3. Observation of imposed fluid restrictions 4. Avoidance of the use of opioids

2. Health teaching to help reduce sickling crises Because there is no cure for sickle cell disease, prevention is one of the main goals of therapeutic management. Thus, health teaching to help reduce sickling crises is key. Anticoagulants do not prevent sickling. Fluids are encouraged to increase the fluid volume and prevent sickling. Opioids usually are needed for pain control.

When providing immediate postoperative care for the child with a cleft palate (CP), the nurse should position the child in which of the following positions? 1. In the supine position 2. In the prone position 3. In an infant seat 4. On his side

2. In the prone position Postoperatively, children with a CP should be placed on their abdomens to facilitate drainage. If the child is placed in the supine position, aspiration is a concern. Using an infant seat does not facilitate drainage. Sidelying does not facilitate drainage as well as the prone position.

Which of the following would not be a focus of a teaching plan for an adolescent with a seizure disorder? 1. Ability to obtain a driver's license 2. Increased risk of infections 3. Drug and alcohol use 4. Peer pressure

2. Increased risk of infections Adolescents with seizure disorders are at no greater risk for infections than other adolescents. The ability to obtain a driver's license may be influenced by the adolescent's seizure history. Drug and alcohol use may interfere with or cause adverse reactions from anticonvulsants. Peer pressure may put the child at risk for increased risk-taking behaviors that may exacerbate seizure activity.

Secondary vesicoureteral reflux usually results from which of the following? 1. Congenital defects 2. Infection 3. Acidic urine 4. Hydronephrosis

2. Infection Infection is the most common cause of secondary vesicoureteral reflux. Congenital defects cause primary vesicoureteral reflux. Acidic urine is normal and helps to prevent infection. Hydronephrosis may result from vesicoureteral reflux.

A child's fasting blood glucose levels range between 100 and 120 mg/dL (5.7 and 6.9 mmol/L) daily. The before-dinner blood glucose levels are between 120 and 130 mg/dL (6.9 and 7.4 mmol/L), with no reported episodes of hypoglycemia. Mixed insulin is administered before breakfast and before dinner. The nurse should make which interpretation about these findings? 1. Exercise should be increased to reduce blood glucose levels. 2. Insulin doses are appropriate for food ingested and activity level. 3. Dietary needs are being met for adequate growth and development. 4. Dietary intake should be increased to avoid hypoglycemic reactions.

2. Insulin doses are appropriate for food ingested and activity level. Blood glucose levels are a measure of the balance among diet, medication, and exercise. Options 1 and 4 imply that the data analyzed are abnormal. The question presents no data for determining growth and development status, such as height, weight, age, or behavior. Supporting normal growth and development is an important goal in managing diabetes in children, but that is not what is being evaluated here. Cognitive Ability: Evaluating Client Needs: Physiological Integrity Integrated Process: Nursing Process: Evaluation Content Area: Pediatrics: Metabolic/Endocrine Strategy(ies): Subject Priority Concepts: Clinical Judgment, Glucose Regulation

A 4-year-old child with leukemia is admitted to the health care facility because of pneumonia. Which of the following is the most likely cause of his current condition? 1. Anemia 2. Leukopenia 3. Thrombocytopenia 4. Eosinophilia

2. Leukopenia Leukopenia, the decrease in functioning white blood cells in leukemia, causes the increased risk of infection in children with leukemia. Anemia would result in fatigue. Thrombocytopenia, decreased platelet count, would result in bleeding. An increased eosinophil count is not related to leukemia.

When developing a teaching plan to prevent urinary tract infection, which of the following should be included? (Select all that apply.) 1. Wearing underwear made of synthetic material such as nylon 2. Maintaining adequate fluid intake 3. Keeping urine alkaline by avoiding acidic beverages 4. Avoiding urination before and after intercourse 5. Avoiding bubble baths and tight clothing 6. Emptying bladder with each urination

2. Maintaining adequate fluid intake 5. Avoiding bubble baths and tight clothing 6. Emptying bladder with each urination Fluid intake helps dilute urine and minimize infection potential, bubble baths and tight clothing may act as irritants, and emptying the bladder fully with each urination prevents stasis. Children and teens should wear cotton underwear, keep their urine acidic, and void before and after intercourse (if sexually active).

A 4-year-old boy is about to be discharged after undergoing surgery and follow-up treatment for a Wilms' tumor. Which of the following points would be a vital part of the teaching program for the child's parents? 1. Allowing him to resume activity, including contact sports 2. Monitoring for signs and symptoms of urinary tract infection 3. Making arrangements for a return visit in 6 months 4. Arranging for hospice care, because Wilms' tumor is fatal

2. Monitoring for signs and symptoms of urinary tract infection Urinary tract infections pose a threat to the remaining kidney. Therefore, the parents should be instructed to monitor the child for signs and symptoms of infection. Rough play and contact sports should be discouraged because of the residual effect of radiation to the abdomen and because the child needs to protect his lone kidney. Follow-up at 6 months is too late; children receive chemotherapy for 6 to 15 months after surgery. Wilms' tumor is the most curable solid tumor of childhood; prognosis is usually favorable.

The nurse is monitoring a child for bleeding after surgery for removal of a brain tumor. The nurse checks the head dressing for the presence of blood and notes a colorless drainage on the back of the dressing. Which intervention should the nurse perform immediately? 1. Reinforce the dressing. 2. Notify the health care provider (HCP). 3. Document the findings and continue to monitor. 4. Circle the area of drainage and continue to monitor.

2. Notify the health care provider (HCP). Colorless drainage on the dressing in a child after craniotomy indicates the presence of cerebrospinal fluid and should be reported to the HCP immediately. Options 1, 3, and 4 are not the immediate nursing intervention because they do not address the need for immediate intervention to prevent complications.

A child undergoes surgical removal of a brain tumor. During the postoperative period, the nurse notes that the child is restless, the pulse rate is elevated, and the blood pressure has decreased significantly from the baseline value. The nurse suspects that the child is in shock. Which is the most appropriate nursing action? 1. Place the child in a supine position. 2. Notify the health care provider (HCP). 3. Place the child in Trendelenburg position. 4. Increase the flow rate of the intravenous fluids.

2. Notify the health care provider (HCP). In the event of shock, the HCP is notified immediately before the nurse changes the child's position or increases intravenous fluids. After craniotomy, a child is never placed in the supine or Trendelenburg position because it increases intracranial pressure (ICP) and the risk of bleeding. The head of the bed should be elevated. Increasing intravenous fluids can cause an increase in ICP.

The nurse is caring for an infant with gastroenteritis who is being treated for dehydration. The nurse reviews the health record and notes that the health care provider has documented that the infant is mildly dehydrated. Which assessment finding should the nurse expect to note in mild dehydration? 1. Anuria 2. Pale skin color 3. Sunken fontanels 4. Dry mucous membranes

2. Pale skin color In mild dehydration the skin color is pale. Anuria and sunken fontanels are assessment characteristics of severe dehydration. Dry mucous membranes are an assessment characteristic of moderate dehydration. Cognitive Ability: Analyzing Client Needs: Physiological Integrity Integrated Process: Nursing Process: Assessment Content Area: Pediatrics: Metabolic/Endocrine Strategy(ies): Subject Priority Concepts: Clinical Judgment, Fluid and Electrolyte Balance

The nurse admits a child to the hospital with a diagnosis of pyloric stenosis. On assessment, which data would the nurse expect to obtain when asking the parent about the child's symptoms? 1. Watery diarrhea 2. Projectile vomiting 3. Increased urine output 4. Vomiting large amounts of bile

2. Projectile vomiting In pyloric stenosis, hypertrophy of the circular muscles of the pylorus causes narrowing of the pyloric canal between the stomach and the duodenum. Clinical manifestations of pyloric stenosis include projectile vomiting, irritability, hunger and crying, constipation, and signs of dehydration, including a decrease in urine output.

Which specific nursing interventions are implemented in the care of a child with leukemia who is at risk for infection? Select all that apply. 1. Maintain the child in a semiprivate room. 2. Reduce exposure to environmental organisms. 3. Use strict aseptic technique for all procedures. 4. Ensure that anyone entering the child's room wears a mask. 5. Apply firm pressure to a needle-stick area for at least 10 minutes.

2. Reduce exposure to environmental organisms. 3. Use strict aseptic technique for all procedures. 4. Ensure that anyone entering the child's room wears a mask. Leukemia is a malignant increase in the number of leukocytes, usually at an immature stage, in the bone marrow. It affects the bone marrow, causing anemia from decreased erythrocytes, infection from neutropenia, and bleeding from decreased platelet production (thrombocytopenia). A common complication of treatment for leukemia is overwhelming infection secondary to neutropenia. Measures to prevent infection include the use of a private room, strict aseptic technique, restriction of visitors and health care personnel with active infection, strict hand washing, ensuring that anyone entering the child's room wears a mask, and reducing exposure to environmental organisms by eliminating raw fruits and vegetables from the diet and fresh flowers from the child's room and by not leaving standing water in the child's room. Applying firm pressure to a needle-stick area for at least 10 minutes is a measure to prevent bleeding.

An infant with a diagnosis of hydrocephalus is scheduled for surgery. Which is the priority nursing intervention in the preoperative period? 1. Test the urine for protein. 2. Reposition the infant frequently. 3. Provide a stimulating environment. 4. Assess blood pressure every 15 minutes.

2. Reposition the infant frequently. Hydrocephalus occurs as a result of an imbalance of cerebrospinal fluid absorption or production that is caused by malformations, tumors, hemorrhage, infections, or trauma. It results in head enlargement and increased intracranial pressure (ICP). In infants with hydrocephalus, the head grows at an abnormal rate, and if the infant is not repositioned frequently, pressure ulcers can occur on the back and side of the head. An egg crate mattress under the head is also a nursing intervention that can help to prevent skin breakdown. Proteinuria is not specific to hydrocephalus. Stimulus should be kept at a minimum because of the increase in ICP. It is not necessary to check the blood pressure every 15 minutes.

The nursing student is presenting a clinical conference and discusses the cause of β-thalassemia. The nursing student informs the group that a child at greatest risk of developing this disorder is which of these? 1. A child of Mexican descent 2.A child of Mediterranean descent 3.A child whose intake of iron is extremely poor 4.A breast-fed child of a mother with chronic anemia

2.A child of Mediterranean descent β-Thalassemia is an autosomal recessive disorder characterized by the reduced production of 1 of the globin chains in the synthesis of hemoglobin (both parents must be carriers to produce a child with β-thalassemia major). This disorder is found primarily in individuals of Mediterranean descent. Options 1, 3, and 4 are incorrect. Cognitive Ability: Applying Client Needs: Physiological Integrity Integrated Process: Teaching and Learning Content Area: Pediatrics: Hematological Strategy(ies): Subject Priority Concepts: Gas Exchange, Perfusion

A child is scheduled to receive inactivitated Polio vaccine (IPV) and the nurse preparing the vaccine reviews the kids chart. The nurse should question this if what is in the kids chart? 1. Recent recovery from cold 2. A history of frequent respiratory infections 3. A history of an anaphyalactic reaction to neomycin 4. A local reaction at the site if injection of a previous IPV

3 *This contains neomycin

A nurse prepares to give a MMR to a 5 yr old. What route should this be given? 1. Sub Q in the gluteal muscle 2. IM in the deltoid 3. subq in the outer aspect of the upper arm 4. IM in the anterolateral aspect of the thigh

3 MMR is given subq in the outer aspect of the upper arm

The nurse is talking to the parents of a child newly diagnosed with diabetes mellitus. Which statement by the parents indicates an understanding of preventing and managing hyperglycemia? 1. I will give 8 oz of diet cola at the first sign of weakness." 2. "I will administer glucagon immediately if shakiness is felt." 3. "I will check for ketones when my child is suffering from an illness." 4. "I will report to the emergency department if the blood glucose level is over 150 mg/dL (8.6 mmol/L)."

3. "I will check for ketones when my child is suffering from an illness." It is recommended that urine be tested for ketones every 3 hours during an illness or whenever the blood glucose level is over 240 mg/dL (13.7 mmol/L) when illness is not present. The child or parents should carry a source of glucose so it is readily available in the event of a hypoglycemic, not hyperglycemic, reaction. A diet carbonated beverage does not meet the need of providing a glucose source during a hypoglycemic episode. If the blood glucose level is greater than 150 mg/dL (8.6 mmol/L), it is unnecessary to report to the emergency department as a first-line treatment. Glucagon is used for an unconscious client who is experiencing a hypoglycemic, not hyperglycemic, reaction and who is unable to swallow. Cognitive Ability: Evaluating Client Needs: Physiological Integrity Integrated Process: Nursing Process: Evaluation Content Area: Pediatrics: Metabolic/Endocrine Strategy(ies): Subject Priority Concepts: Client Education, Glucose Regulation

The nurse is preparing to administer an MMR (measles, mumps, and rubella) vaccine to a 15-month-old child. Before administering the vaccine, which question should the nurse ask the mother of the child? 1. "Has the child had any sore throats?" 2. "Has the child been eating properly?" 3. "Is the child allergic to any antibiotics?" 4. "Has the child been exposed to any infections?"

3. "Is the child allergic to any antibiotics?" Before the administration of MMR vaccine, a thorough health history needs to be obtained. MMR is used with caution in a child with a history of an allergy to gelatin, eggs, or neomycin because the live measles vaccine is produced by chick embryo cell culture and because MMR also contains a small amount of the antibiotic neomycin. The questions in the remaining options are not directed at addressing contraindications to administering immunizations. Cognitive Ability: Analyzing Client Needs: Physiological Integrity Integrated Process: Nursing Process: Assessment Content Area: Pediatrics: Immune Strategy(ies): Subject Priority Concepts: Health Promotion, Safety

A 6-year-old child with leukemia is hospitalized and is receiving combination chemotherapy. Laboratory results indicate that the child is neutropenic, and protective isolation procedures are initiated. The grandmother of the child visits and brings a fresh bouquet of flowers picked from her garden, and asks the nurse for a vase for the flowers. Which response should the nurse provide to the grandmother? 1. "I have a vase in the utility room, and I will get it for you." 2. "I will get the vase and wash it well before you put the flowers in it." 3. "The flowers from your garden are beautiful, but should not be placed in the child's room at this time." 4. "When you bring the flowers into the room, place them on the bedside stand as far away from the child as possible."

3. "The flowers from your garden are beautiful, but should not be placed in the child's room at this time." Leukemia is a malignant increase in the number of leukocytes, usually at an immature stage, in the bone marrow. It affects the bone marrow, causing anemia from decreased erythrocytes, infection from neutropenia, and bleeding from decreased platelet production (thrombocytopenia). For a hospitalized neutropenic child, flowers or plants should not be kept in the room because standing water and damp soil harbor Aspergillus and Pseudomonas aeruginosa, to which the child is susceptible. In addition, fresh fruits and vegetables harbor molds and should be avoided until the white blood cell count increases.

The parents of a child recently diagnosed with cerebral palsy ask the nurse about the limitations of the disorder. The nurse responds by explaining that the limitations occur as a result of which pathophysiological process? 1. An infectious disease of the central nervous system 2. An inflammation of the brain as a result of a viral illness 3. A chronic disability characterized by impaired muscle movement and posture 4. A congenital condition that results in moderate to severe intellectual disabilities

3. A chronic disability characterized by impaired muscle movement and posture Cerebral palsy is a chronic disability characterized by impaired movement and posture resulting from an abnormality in the extrapyramidal or pyramidal motor system. Meningitis is an infectious process of the central nervous system. Encephalitis is an inflammation of the brain that occurs as a result of viral illness or central nervous system infection. Down syndrome is an example of a congenital condition that results in moderate to severe intellectual disabilities.

The nurse is caring for a child with a diagnosis of hemophilia, and hemarthrosis is suspected because the child is complaining of pain in the joints. Which measure should the nurse expect to be prescribed for the child? 1. Range-of-motion exercises to the affected joint 2. Application of a heating pad to the affected joint 3. Application of a bivalved cast for joint immobilization 4. Nonsteroidal antiinflammatory drugs for the pain

3. Application of a bivalved cast for joint immobilization In an acute period, immobilization of the joint would be prescribed. Range-of-motion exercise during the acute period can increase the bleeding and would be avoided at this time. Heat will increase blood flow to the area, so it would promote increased bleeding to the area. Nonsteroidal antiinflammatory drugs (NSAIDs) can prolong bleeding time and would not be prescribed for the child. Cognitive Ability: Analyzing Client Needs: Physiological Integrity Integrated Process: Nursing Process: Planning Content Area: Pediatrics: Hematological Strategy(ies): Subject Priority Concepts: Clinical Judgment, Clotting

The nurse collects a urine specimen preoperatively from a child with epispadias who is scheduled for surgical repair. When analyzing the results of the urinalysis, which should the nurse most likely expect to note? 1. Hematuria 2. Proteinuria 3. Bacteriuria 4. Glucosuria

3. Bacteriuria Epispadias is a congenital defect involving abnormal placement of the urethral orifice of the penis. The urethral opening is located anywhere on the dorsum of the penis. This anatomical characteristic facilitates entry of bacteria into the urine. Options 1, 2, and 4 are not characteristically noted in this condition.

The nurse reviews the record of a newborn infant and notes that a diagnosis of esophageal atresia with tracheoesophageal fistula is suspected. The nurse expects to note which most likely sign of this condition documented in the record? 1. Incessant crying 2. Coughing at nighttime 3. Choking with feedings 4. Severe projectile vomiting

3. Choking with feedings In esophageal atresia and tracheoesophageal fistula, the esophagus terminates before it reaches the stomach, ending in a blind pouch, and a fistula is present that forms an unnatural connection with the trachea. Any child who exhibits the "3 Cs"—coughing and choking with feedings and unexplained cyanosis—should be suspected to have tracheoesophageal fistula. Options 1, 2, and 4 are not specifically associated with tracheoesophageal fistula.

A lumbar puncture is performed on a child suspected to have bacterial meningitis, and cerebrospinal fluid (CSF) is obtained for analysis. The nurse reviews the results of the CSF analysis and determines that which results would verify the diagnosis? 1. Clear CSF, decreased pressure, and elevated protein level 2. Clear CSF, elevated protein, and decreased glucose levels 3. Cloudy CSF, elevated protein, and decreased glucose levels 4. Cloudy CSF, decreased protein, and decreased glucose levels

3. Cloudy CSF, elevated protein, and decreased glucose levels Meningitis is an infectious process of the central nervous system caused by bacteria and viruses; it may be acquired as a primary disease or as a result of complications of neurosurgery, trauma, infection of the sinus or ears, or systemic infections. Meningitis is diagnosed by testing CSF obtained by lumbar puncture. In the case of bacterial meningitis, findings usually include an elevated pressure; turbid or cloudy CSF; and elevated leukocyte, elevated protein, and decreased glucose levels.

The nurse is caring for a newborn with a suspected diagnosis of imperforate anus. The nurse monitors the infant, knowing that which is a clinical manifestation associated with this disorder? 1. Bile-stained fecal emesis 2. The passage of currant jelly-like stools 3. Failure to pass meconium stool in the first 24 hours after birth 4. Sausage-shaped mass palpated in the upper right abdominal quadrant

3. Failure to pass meconium stool in the first 24 hours after birth Imperforate anus is the incomplete development or absence of the anus in its normal position in the perineum. During the newborn assessment, this defect should be identified easily on sight. However, a rectal thermometer or tube may be necessary to determine patency if meconium is not passed in the first 24 hours after birth. Other assessment findings include absence or stenosis of the anal rectal canal, presence of an anal membrane, and an external fistula to the perineum. Options 1, 2, and 4 are findings noted in intussusception.

When assessing a child for possible intussusception, which of the following would be least likely to provide valuable information? 1. Stool inspection 2. Pain pattern 3. Family history 4. Abdominal palpation

3. Family history Because intussusception is not believed to have a familial tendency, obtaining a family history would provide the least amount of information. Stool inspection, pain pattern, and abdominal palpation would reveal possible indicators of intussusception. "Currant jelly" stools, containing blood and mucus, are an indication of intussusception. Acute, episodic abdominal pain is characteristic of intussusception. A sausage-shaped mass may be palpated in the right upper quadrant.

A 6-year-old child with diabetes mellitus and the child's mother come to the health care clinic for a routine examination. The nurse evaluates the data collected during this visit to determine if the child has been euglycemic since the last visit. Which information is the most significant indicator of euglycemia? 1. Daily glucose monitor log 2. Dietary history for the previous week 3. Glycosylated hemoglobin (hemoglobin A1c) 4. Fasting blood glucose performed on the day of the clinic visit

3. Glycosylated hemoglobin (hemoglobin A1c) The glycosylated hemoglobin assay measures the glucose molecules that attach to the hemoglobin A molecules and remain there for the life of the red blood cell, approximately 120 days. This is not reversible and cannot be altered by human intervention. Daily glucose logs are useful if they are kept regularly and accurately. However, they reflect only the blood glucose at the time the test was done. A fasting blood glucose test performed on the day of the clinic visit is time limited in its scope, as is the dietary history. Cognitive Ability: Evaluating Client Needs: Physiological Integrity Integrated Process: Nursing Process: Evaluation Content Area: Pediatrics: Metabolic/Endocrine Strategy(ies): Subject, Strategic Words Priority Concepts: Clinical Judgment, Glucose Regulation

Which of the following is considered the most common assessment finding associated with a brain tumor in a child? 1. Projectile vomiting 2. Increased intracranial pressure (ICP) 3. Headache 4. Diminished reflexes

3. Headache In children with a brain tumor, the most common symptom is a headache. It is typically described as intermittent and most common after the child wakes up. It may also be described as occurring with sneezing, coughing, and straining during a bowel movement. Vomiting, which usually is projectile and occurs in the morning, is not as common, but it may be present. The tumor as it increases in size obstructs the circulation of cerebrospinal fluid, resulting in increased ICP. Decreased reflexes are commonly reported in cerebellar tumors.

A nursing student is assigned to care for a child with sickle cell disease (SCD). The nursing instructor asks the student to describe the causative factors related to this disease. Which statement by the student indicates a need for further research? 1. SCD is an autosomal recessive disease. 2. Children with the HbS (sickle cell hemoglobin) trait are not symptomatic. 3. If each parent carries the trait, the child will carry the trait, and the probability of the child having the disease is 75%. 4. If one parent has the HbS trait and the other parent is normal, there is a 50% chance that each offspring will inherit the trait.

3. If each parent carries the trait, the child will carry the trait, and the probability of the child having the disease is 75%. SCD is an autosomal recessive disease. Children with the HbS trait are not symptomatic. If one parent has the HbS trait and the other parent is normal, there is a 50% chance that each offspring will inherit the trait. If each parent carries the trait, there is a 25% chance that their child will be normal, a 50% chance that the child will carry the trait, and a 25% chance that each child will have the disease. Cognitive Ability: Evaluating Client Needs: Physiological Integrity Integrated Process: Teaching and Learning Content Area: Pediatrics: Hematological Strategy(ies): Negative Event Query, Strategic Words Priority Concepts: Cellular Regulation, Clinical Judgment

An infant has just returned to the nursing unit after surgical repair of a cleft lip on the right side. The nurse should place the infant in which best position at this time? 1. Prone position 2. On the stomach 3. Left lateral position 4. Right lateral position

3. Left lateral position A cleft lip is a congenital anomaly that occurs as a result of failure of soft tissue or bony structure to fuse during embryonic development. After cleft lip repair, the nurse avoids positioning an infant on the side of the repair or in the prone position because these positions can cause rubbing of the surgical site on the mattress. The nurse positions the infant on the side lateral to the repair or on the back upright and positions the infant to prevent airway obstruction by secretions, blood, or the tongue. From the options provided, placing the infant on the left side immediately after surgery is best to prevent the risk of aspiration if the infant vomits.

Which of the following is the most common form of childhood cancer? 1. Lymphoma 2. Brain tumors 3. Leukemia 4. Osteosarcoma

3. Leukemia Leukemia is the most common type of cancer in children, followed by brain tumors, lymphoma, and kidney tumors. Brain tumors are the second most common childhood cancer, but they are the most common form of solid tumor cancer in childhood. Bone cancers account for 5%, with osteosarcoma being the most common type.

In children with sickle cell disease, tissue damage results from which of the following? 1. A general inflammatory response due to an autoimmune reaction from hypoxia 2. Air hunger and respiratory alkalosis due to deoxygenated red blood cells 3. Local tissue damage with ischemia and necrosis due to obstructed circulation 4. Hypersensitivity of the central nervous system (CNS) due to elevated serum bilirubin levels

3. Local tissue damage with ischemia and necrosis due to obstructed circulation Characteristic sickle cells tend to clump, which results in poor circulation to tissue, local tissue damage, and eventual ischemia and necrosis. In sickle cell anemia, damage is not due to an inflammatory response. Air hunger and respiratory alkalosis are not present. The CNS effects result from ischemia.

A child is hospitalized because of persistent vomiting. The nurse should monitor the child closely for which problem? 1. Diarrhea 2. Metabolic acidosis 3. Metabolic alkalosis 4. Hyperactive bowel sounds

3. Metabolic alkalosis Vomiting causes the loss of hydrochloric acid and subsequent metabolic alkalosis. Metabolic acidosis would occur in a child experiencing diarrhea because of the loss of bicarbonate. Diarrhea might or might not accompany vomiting. Hyperactive bowel sounds are not associated with vomiting.

Which of the following signs and symptoms are characteristic of minimal-change nephrotic syndrome? 1. Gross hematuria, proteinuria, fever 2. Hypertension, edema, hematuria 3. Poor appetite, proteinuria, edema 4. Hypertension, edema, proteinuria

3. Poor appetite, proteinuria, edema Clinical manifestations of nephrotic syndrome include loss of appetite due to edema of intestinal mucosa, proteinuria, and edema. Gross hematuria is not associated with nephrotic syndrome. Fever would occur only if infection also existed. Hypertension alone or accompanied by hematuria is associated with glomerulonephritis.

When performing a procedure related to a genitourinary (GU) problem, the nurse would anticipate that which of the following age groups would find it especially stressful? 1. Infants 2. Toddlers 3. Preschoolers 4. School-age children

3. Preschoolers In general, preschoolers have more fears because of their fantasies, contributing to fears of the simplest procedures. Castration fears also are prominent at this age and may be heightened by procedures related to GU problems. Typically, GU procedures do not create greater stress in infants, toddlers, and school-age children.

When caring for a child with leukemia, which of the following goals should be considered primary? 1. Meeting developmental needs 2. Promoting adequate nutrition 3. Preventing infection 4. Promoting diversionary activities

3. Preventing infection The child is at high risk for infection due to immunosuppression from both the disease and the treatment, and infection is the leading cause of death in leukemia. Meeting developmental needs, promoting adequate nutrition, and promoting diversionary activities are important goals. However, they are not the primary goal in caring for a child with leukemia.

While assessing a child with pyloric stenosis, the nurse is likely to note which of the following? 1. Regurgitation 2. Steatorrhea 3. Projectile vomiting 4. "Currant jelly" stools

3. Projectile vomiting Projectile vomiting is a key sign of pyloric stenosis. Regurgitation is seen more commonly with gastroesophageal reflux. Steatorrhea occurs in malabsorption disorders such as celiac disease. "Currant jelly" stools are characteristic of intussusception.

The nurse is reviewing the record of a child with increased intracranial pressure and notes that the child has exhibited signs of decerebrate posturing. On assessment of the child, the nurse expects to note which characteristic of this type of posturing? 1. Flaccid paralysis of all extremities 2. Adduction of the arms at the shoulders 3. Rigid extension and pronation of the arms and legs 4. Abnormal flexion of the upper extremities and extension and adduction of the lower extremities

3. Rigid extension and pronation of the arms and legs Decerebrate (extension) posturing is characterized by the rigid extension and pronation of the arms and legs. Option 1 is incorrect. Options 2 and 4 describe decorticate (flexion) posturing.

Which of the following definitions most accurately describes meningocele? 1. Complete exposure of the spinal cord and meninges 2. Herniation of the spinal cord and meninges into a sac 3. Sac formation containing meninges and spinal fluid 4. Spinal cord tumor containing nerve roots

3. Sac formation containing meninges and spinal fluid Meningocele is a sac formation containing meninges and cerebrospinal fluid (CSF). Meningocele doesn't involve complete exposure of the spinal cord and meninges; this is a massive defect that's incompatible with life. Myelomeningocele is a herniation of the spinal cord, meninges, and CSF into a sac that protrudes through a defect in the vertebral arch. Tumor formation is not associated with this defect.

When teaching parents about the early signs and symptoms of lead toxicity, which of the following if stated by the parents would indicate the need for additional teaching? 1. Anorexia 2. Irritability 3. Seizures 4. Anemia

3. Seizures Seizures usually are associated with encephalopathy, a late sign of lead toxicity. Typically, lead levels have already exceeded 70 mg/dl. Anorexia, irritability, and anemia are early signs.

The nurse is providing home care instructions to the parents of a 10-year-old child with hemophilia. Which sport activity should the nurse suggest for this child? 1. Soccer 2. Basketball 3. Swimming 4. Field hockey

3. Swimming Children with hemophilia need to avoid contact sports and to take precautions such as wearing elbow and knee pads and helmets with other sports. The safe activity for them is swimming. Cognitive Ability: Applying Client Needs: Safe and Effective Care Environment Integrated Process: Teaching and Learning Content Area: Pediatrics: Hematological Strategy(ies): Comparable or Alike Options, Subject Priority Concepts: Clotting, Safety

The nurse is collecting data on a 12-month-old child with iron deficiency anemia. Which finding should the nurse expect to note in this child? 1. Cyanosis 2. Bronze skin 3. Tachycardia 4. Hyperactivity

3. Tachycardia Clinical manifestations of iron deficiency anemia will vary with the degree of anemia but usually include extreme pallor with a porcelain-like skin, tachycardia, lethargy, and irritability. Cognitive Ability: Analyzing Client Needs: Physiological Integrity Integrated Process: Nursing Process: Assessment Content Area: Pediatrics: Hematological Strategy(ies): Subject Priority Concepts: Clinical Judgment, Gas Exchange

An adolescent is examined in the hospital emergency department after taking an overdose of acetylsalicylic acid. The adolescent has rapid breathing, nausea and vomiting, and lethargy. The health care provider prescribes arterial blood specimens for blood gas analysis to be drawn. Aspirin toxicity is suspected when the blood gas results are reported as which value? 1. pH 7.50, Pco2 60 mmHg, HCO3 30 mEq/L (30 mmol/L) 2. pH 7.44, Pco2 30 mmHg, HCO3 21 mEq/L (21 mmol/L) 3. pH 7.29, Pco2 29 mmHg, HCO3 19 mEq/L (19 mmol/L) 4. pH 7.33, Pco2 52 mmHg, HCO3 28 mEq/L (28 mmol/L)

3. pH 7.29, Pco2 29 mmHg, HCO3 19 mEq/L (19 mmol/L) The client who has aspirin toxicity will manifest metabolic acidosis with respiratory compensation as seen when the pH is lower 7.35 mm Hg and the HCO3 is less than 22 mEq/L (22 mmol/L). In the correct option, the pH is acidotic and the HCO3 is decreased, indicating metabolic acidosis. The Pco2 is alkalotic, indicating partial compensation. Cognitive Ability: Analyzing Client Needs: Physiological Integrity Integrated Process: Nursing Process: Analysis Content Area: Pediatrics: Metabolic/Endocrine Strategy(ies): Subject Priority Concepts: Acid-Base Balance, Clinical Judgment

The mother of a 6-year-old child who has type 1 diabetes mellitus calls a clinic nurse and tells the nurse that the child has been sick. The mother reports that she checked the child's urine and it was positive for ketones. The nurse should instruct the mother to take which action? 1.Hold the next dose of insulin. 2.Come to the clinic immediately. 3.Encourage the child to drink liquids. 4.Administer an additional dose of regular insulin

3.Encourage the child to drink liquids. When the child is sick, the mother should test for urinary ketones with each voiding. If ketones are present, liquids are essential to aid in clearing the ketones. The child should be encouraged to drink liquids. Bringing the child to the clinic immediately is unnecessary. Insulin doses should not be adjusted or changed. Cognitive Ability: Applying Client Needs: Physiological Integrity Integrated Process: Nursing Process: Implementation Content Area: Pediatrics: Metabolic/Endocrine Strategy(ies): Comparable or Alike Options Priority Concepts: Clinical Judgment, Glucose Regulation

A kid is receiving a series of Hep B vaccines and arrives for a second dose. Before giving the nurse should ask the kid and parent about a history of a severe allergic reaction to what? 1. Eggs 2. PCN 3. Sulfonamides 4. A previous dose of HEP B vaccine or component

4 * A contraindication is a previous reaction to a previous dose of HEP B or a component ( aluminum hydroxide or yeast protein)

A nurse visits a child with Mono and provides care instructions to the parents. Which instruction should the nurse give the parents? 1. maintain bed rest for 2 weeks 2. maintain Resp. precautions for 1 week 3. Notify HCP if kid develops a fever 4. Notify HCP if child develops ABD pain Left shoulder pain

4 * Mono is caused by Epstein- Barr virus. Parents need to report pain in ABD, especially in LUQ or left shoulder pain ; this may indicate that the spleen has ruptured.

The nurse provides home care instructions to the parents of a child hospitalized with pertussis who is in the convalescent stage and is being prepared for d/c. What statement made by parent indicates a NEED for further instruction? 1. We need to encourage our child to drink fluids 2. Coughing spells may be triggered by dust or smoke 3. Vomiting may occur when our child has coughing episodes 4. We need to maintain droplet precautions and a quiet environment for at leat 2 weeks

4 * pertussis is transmitted by direct contact or resp. droplets from coughing

A 4 month old comes to clinic for shots. The child is up to date. What vaccines should the kid get at this time? 1. varicella, HEP B 2. DTaP, MMR, IPV 3. MMR, Hib, DTaP 4. DTaP, hib, PCV, RV

4 DTap, hib, PVC and RV are given at 4 months

An infant receives a a DTaP immuinization at a well baby clinic. The parent returns home and calls the clinic and reports swelling & redness at the injection site. Which is an appropriate action? 1. Monitor the infant for a fever 2. Bring the infant back to clinic 3. Apply hot pack 4. Apply cold pack

4 * On occasion tenderness, swelling and redness appear at the site of a DTaP injection. Cold packs for the first 24 hrs, followed by cold or warm compresses if inflammation persist.

howlongdoes Vaso Occlusive crisislastfor?

4-6 days sickle cells block blood flow and decrease oxygen delivery

The nurse is reviewing a treatment plan with the parents of a newborn with hypospadias. Which statement by the parents indicates their understanding of the plan? 1. "Caution should be used when straddling the infant on a hip." 2. "Vital signs should be taken daily to check for bladder infection." 3. "Catheterization will be necessary when the infant does not void." 4. "Circumcision has been delayed to save tissue for surgical repair."

4. "Circumcision has been delayed to save tissue for surgical repair." Hypospadias is a congenital defect involving abnormal placement of the urethral orifice of the penis. In hypospadias, the urethral orifice is located below the glans penis along the ventral surface. The infant should not be circumcised because the dorsal foreskin tissue will be used for surgical repair of the hypospadias. Options 1, 2, and 3 are unrelated to this disorder.

Which question should the nurse ask the parents of a child suspected of having glomerulonephritis? 1. "Did your child fall off a bike onto the handlebars?" 2. "Has the child had persistent nausea and vomiting?" 3. "Has the child been itching or had a rash anytime in the last week?" 4. "Has the child had a sore throat or a throat infection in the last few weeks?"

4. "Has the child had a sore throat or a throat infection in the last few weeks?" Glomerulonephritis refers to a group of kidney disorders characterized by inflammatory injury in the glomerulus. Group A β-hemolytic streptococcal infection is a cause of glomerulonephritis. Often, a child becomes ill with streptococcal infection of the upper respiratory tract and then develops symptoms of acute poststreptococcal glomerulonephritis after an interval of 1 to 2 weeks. The assessment data in options 1, 2, and 3 are unrelated to a diagnosis of glomerulonephritis.

A home care nurse is teaching an adolescent with type 1 diabetes mellitus about insulin administration and rotation sites. Which statement, if made by the adolescent, would indicate effective teaching? 1. "I should use only my stomach and my thighs for injections." 2. "I need to use a different major site for each insulin injection." 3. "I should use the same major site for 1 month before rotating to another site." 4. "I need to give 4 to 6 injections in one area, about an inch apart, and then move to another area."

4. "I need to give 4 to 6 injections in one area, about an inch apart, and then move to another area." The most efficient rotation plan involves giving about four to six injections in one area, each injection about 1 inch (2.5 cm) apart, or the diameter of the insulin vial from the previous injection, and then moving to another area of the major site the next day. All other options are incorrect. Cognitive Ability: Evaluating Client Needs: Physiological Integrity Integrated Process: Nursing Process: Evaluation Content Area: Pediatrics: Metabolic/Endocrine Strategy(ies): Closed-ended Word, Strategic Words Priority Concepts: Client Education, Glucose Regulation

Which of the following would the nurse expect to assess as the most common presenting manifestation of Wilms' tumor? 1. Hematuria 2. Pain on voiding 3. Nausea and vomiting 4. Abdominal mass

4. Abdominal mass The most common sign of Wilms' tumor is a painless palpable mass, sometimes marked by an increase in abdominal girth. Gross hematuria is uncommon, although microscopic hematuria may be present. Pain on voiding is not associated with Wilms' tumor. Tumor encroachment should not cause abdominal obstruction and resulting nausea and vomiting.

Which of the following nursing interventions can help prevent or reduce nausea and vomiting during chemotherapy? 1. Providing a high-fiber diet before chemotherapy 2. Administering allopurinol 30 minutes before chemotherapy 3. Encouraging the child to increase his intake of fluid before and during therapy 4. Administering an antiemetic 30 minutes before chemotherapy

4. Administering an antiemetic 30 minutes before chemotherapy Antiemetics counteract nausea most effectively when given before administration of an agent that causes nausea. Antiemetics also work best when given on a continuous basis, rather than as needed. A high- fiber diet has no effect on reducing nausea and vomiting. Allopurinol, a xanthine oxidase inhibitor, is thought to prevent renal damage due to large releases of uric acid during chemotherapy; however, it has no antiemetic properties. High fluid intake during periods of nausea exaggerates the symptoms and may exacerbate vomiting.

Which of the following would the nurse identify as normal when assessing infantile reflexes in a 9-month-old? 1. Absent Moro reflex 2. Unilateral grasp 3. Persistent rooting 4. Bilateral parachute

4. Bilateral parachute The parachute reflex appears at about 9 months of age and is normal. All of the following are considered abnormal when evaluating infantile reflexes: reflexes that are absent when they should be present (Moro), reflexes that are unilateral (grasp), and reflexes that persist after they should have disappeared (rooting).

For a child with recurring nephrotic syndrome, which of the following areas of potential disturbances should be a prime consideration when planning ongoing nursing care? 1. Muscle coordination 2. Sexual maturation 3. Intellectual development 4. Body image

4. Body image Because of the edema associated with nephrotic syndrome, potential self-concept and body image disturbances related to changes in appearance and social isolation should be considered. Muscle coordination, sexual maturation, and intellectual function are not affected.

A mother arrives at the emergency department with her 5-year-old child and states that the child fell off a bunk bed. A head injury is suspected. The nurse checks the child's airway status and assesses the child for early and late signs of increased intracranial pressure (ICP). Which is a late sign of increased ICP? 1. Nausea 2. Irritability 3. Headache 4. Bradycardia

4. Bradycardia Head injury is the pathological result of any mechanical force to the skull, scalp, meninges, or brain. A head injury can cause bleeding in the brain and result in increased ICP. In a child, early signs include a slight change in level of consciousness, headache, nausea, vomiting, visual disturbances (diplopia), and seizures. Late signs of increased ICP include a significant decrease in level of consciousness, bradycardia, decreased motor and sensory responses, alterations in pupil size and reactivity, posturing, Cheyne-Stokes respirations, and coma.

The nurse is preparing to care for a child with a diagnosis of intussusception. The nurse reviews the child's record and expects to note which sign of this disorder documented? 1. Watery diarrhea 2. Ribbon-like stools 3. Profuse projectile vomiting 4. Bright red blood and mucus in the stools

4. Bright red blood and mucus in the stools Intussusception is a telescoping of 1 portion of the bowel into another. The condition results in an obstruction to the passage of intestinal contents. A child with intussusception typically has severe abdominal pain that is crampy and intermittent, causing the child to draw in the knees to the chest. Vomiting may be present, but is not projectile. Bright red blood and mucus are passed through the rectum and commonly are described as currant jelly-like stools. Watery diarrhea and ribbon-like stools are not manifestations of this disorder.

A 2-year-old child has just been diagnosed with type 1 diabetes. The nurse is providing education to the parents regarding signs of hypoglycemia. Which of the following information should the nurse include in her teaching session? 1. Child's breath will smell like fruit. 2. Child will complain of excessive thirst. 3. Child will complain of sleepiness and will appear fatigued. 4. Child's behavior will resemble a burst of anger or a temper tantrum.

4. Child's behavior will resemble a burst of anger or a temper tantrum. Caring for toddlers with type 1 diabetes can be difficult because the children's daily behaviors often mimic signs of hypoglycemia. For that reason, parents must be forewarned to consider hypoglycemia as the reason for a child's aberrant behavior rather than simply as a "phase that the child is going through."

After teaching the parents about their child's unique psychological needs related to a seizure disorder and possible stressors, which of the following concerns voiced by them would indicate the need for additional teaching? 1. Poor self-image 2. Dependency 3. Feeling different from peers 4. Cognitive delays

4. Cognitive delays Children with seizure disorders do not necessarily have cognitive delays. Poor self-image, dependency, and feelings of being different from peers can put additional stress on a child trying to understand and manage chronic illness.

A child with β-thalassemia is receiving long-term blood transfusion therapy for the treatment of the disorder. Chelation therapy is prescribed as a result of too much iron from the transfusions. Which medication should the nurse anticipate to be prescribed? 1. Fragmin 2. Meropenem 3. Metoprolol 4. Deferoxamine

4. Deferoxamine The major complication of long-term transfusion therapy is hemosiderosis. To prevent organ damage from too much iron, chelation therapy with either Exjade or deferoxamine may be prescribed. Deferoxamine is classified as an antidote for acute iron toxicity. Fragmin is an anticoagulant used as prophylaxis for postoperative deep vein thrombosis. Meropenem is an antibiotic. Metoprolol is a beta blocker used to treat hypertension. Cognitive Ability: Analyzing Client Needs: Physiological Integrity Integrated Process: Nursing Process: Planning Content Area: Pediatrics: Hematological Strategy(ies): Subject Priority Concepts: Clinical Judgment, Gas Exchange

A 12-year-old child with newly diagnosed thalassemia is brought to the clinic exhibiting delayed sexual maturation, fatigue, anorexia, pallor, and complaints of headache. The child seems listless and small for age and has frontal bossing. What should the nurse expect to note on review of the results of the laboratory tests? 1. Macrocytosis and hyperchromia 2. Excessive red blood cell production 3. Excessive mature erythrocyte proliferation 4. Deficient production of functional hemoglobin

4. Deficient production of functional hemoglobin Defective hemoglobin is produced as a result of genetically deficient beta-polypeptide. This hemoglobin is unstable, disintegrates, and damages the erythrocytes. Rapid destruction of the red cells stimulates rapid production of immature red cells, and the net gain is less than optimally functioning red cells. Iron from the red blood cell destruction is stored in the tissues, causing multiple problems. In thalassemia, immature erythrocytes proliferate, not mature ones. This is a progressive anemia. The nurse also would note microcytosis and hypochromia. Cognitive Ability: Analyzing Client Needs: Physiological Integrity Integrated Process: Nursing Process: Assessment Content Area: Pediatrics: Hematological Strategy(ies): Subject Priority Concepts: Cellular Regulation, Gas Exchange

What is the most likely underlying pathophysiology of primary enuresis? 1. Urinary tract infection 2. Psychogenic stress 3. Vesicoureteral reflux 4. Delayed bladder maturation

4. Delayed bladder maturation The most likely cause of PE is delayed or incomplete maturation of the bladder. UTIs may cause either primary or secondary enuresis, but they are not the leading cause of PE. Psychogenic stress may cause either primary or secondary enuresis, but it is not the leading cause of PE. Vesicoureteral reflux may cause either primary or secondary enuresis, but it is not the leading cause of PE.

Early clinical manifestations of increased intracranial pressure (ICP) in older children include which of the following? 1. Macewen's sign 2. Setting sun sign 3. Papilledema 4. Diplopia

4. Diplopia Diplopia is an early sign of increased ICP in an older child. The Macewen's sign (cracked-pot sound) and the setting sun appearance of the eyes are noted in infants with increased ICP. Papilledema is a late sign of increased ICP.

Which of the following instructions would be included in a sexually active adolescent's preventive teaching plan about urinary tract infections? 1. Wiping back to front 2. Wearing nylon underwear 3. Avoiding urinating before intercourse 4. Drinking acidic juices

4. Drinking acidic juices Drinking acidic juices, such as cranberry juice, helps keep the urine at its desired acid pH and reduces the chance of infection. The client should wipe from front to back, wear cotton underwear, and void before and after intercourse.

Which of the following would the nurse expect when assessing a child with cystitis? 1. High fever 2. Flank pain 3. Costovertebral tenderness 4. Dysuria

4. Dysuria Dysuria is a symptom of a lower urinary tract infection (UTI) such as cystitis. High fever, flank pain, and costovertebral tenderness are signs and symptoms of pyelonephritis, an upper UTI.

A school-age child with type 1 diabetes mellitus has soccer practice and the school nurse provides instructions regarding how to prevent hypoglycemia during practice. Which should the school nurse tell the child to do? 1. Eat twice the amount normally eaten at lunchtime. 2. Take half the amount of prescribed insulin on practice days. 3. Take the prescribed insulin at noontime rather than in the morning. 4. Eat a small box of raisins or drink a cup of orange juice before soccer practice.

4. Eat a small box of raisins or drink a cup of orange juice before soccer practice. Hypoglycemia is a blood glucose level less than 70 mg/dL (4 mmol/L) and results from too much insulin, not enough food, or excessive activity. An extra snack of 15 to 30 g of carbohydrates eaten before activities such as soccer practice would prevent hypoglycemia. A small box of raisins or a cup of orange juice provides 15 to 30 g of carbohydrates. The child or parents should not be instructed to adjust the amount or time of insulin administration. Meal amounts should not be doubled. Cognitive Ability: Applying Client Needs: Physiological Integrity Integrated Process: Teaching and Learning Content Area: Pediatrics: Metabolic/Endocrine Strategy(ies): Comparable or Alike Options Priority Concepts: Glucose Regulation, Health Promotion

Which of the following organisms is the most common cause of urinary tract infection (UTI) in children? 1. Staphylococcus 2. Klebsiella 3. Pseudomonas 4. Escherichia coli

4. Escherichia coli E. coli is the most common organism associated with the development of UTI. Although Staphylococcus, Klebsiella, and Pseudomonas species may cause UTIs, the incidence of UTIs related to each is less than that for E. coli.

The clinic nurse instructs parents of a child with sickle cell anemia about the precipitating factors related to sickle cell crisis. Which, if identified by the parents as a precipitating factor, indicates the need for further instruction? 1. Stress 2. Trauma 3. Infection 4. Fluid overload

4. Fluid overload Sickle cell crises are acute exacerbations of the disease, which vary considerably in severity and frequency; these include vaso-occlusive crisis, splenic sequestration, hyperhemolytic crisis, and aplastic crisis. Sickle cell crisis may be precipitated by infection, dehydration, hypoxia, trauma, or physical or emotional stress. The mother of a child with sickle cell disease should encourage fluid intake of 1½ to 2 times the daily requirement to prevent dehydration.

The clinic nurse instructs parents of a child with sickle cell anemia about the precipitating factors related to sickle cell crisis. Which, if identified by the parents as a precipitating factor, indicates the need for further instruction? 1. Stress 2. Trauma 3. Infection 4. Fluid overload

4. Fluid overload Sickle cell crises are acute exacerbations of the disease, which vary considerably in severity and frequency; these include vaso-occlusive crisis, splenic sequestration, hyperhemolytic crisis, and aplastic crisis. Sickle cell crisis may be precipitated by infection, dehydration, hypoxia, trauma, or physical or emotional stress. The mother of a child with sickle cell disease should encourage fluid intake of 1½ to 2 times the daily requirement to prevent dehydration. Cognitive Ability: Evaluating Client Needs: Health Promotion and Maintenance Integrated Process: Teaching and Learning Content Area: Pediatrics: Hematological Strategy(ies): Negative Event Query, Strategic Words Priority Concepts: Client Education, Gas Exchange

The clinic nurse reviews the record of an infant and notes that the health care provider has documented a diagnosis of suspected Hirschsprung's disease. The nurse reviews the assessment findings documented in the record, knowing that which sign most likely led the mother to seek health care for the infant? 1. Diarrhea 2. Projectile vomiting 3. Regurgitation of feedings 4. Foul-smelling ribbon-like stools

4. Foul-smelling ribbon-like stools Rationale: Hirschsprung's disease is a congenital anomaly also known as congenital aganglionosis or aganglionic megacolon. It occurs as the result of an absence of ganglion cells in the rectum and other areas of the affected intestine. Chronic constipation beginning in the first month of life and resulting in pellet-like or ribbon-like stools that are foul-smelling is a clinical manifestation of this disorder. Delayed passage or absence of meconium stool in the neonatal period is also a sign. Bowel obstruction, especially in the neonatal period; abdominal pain and distention; and failure to thrive are also clinical manifestations. Options 1, 2, and 3 are not associated specifically with this disorder.

Which of the following would be an appropriate nursing diagnosis for a child who is receiving chemotherapy? 1. Ineffective breathing pattern 2. Constipation 3. Impaired skin integrity 4. Impaired oral mucous membrane

4. Impaired oral mucous membrane Chemotherapy destroys rapidly growing cells, including those in the GI tract, resulting in stomatitis, an inflammatory condition of the mouth. In general, chemotherapy should have no effect on the respiratory system. Because chemotherapy affects the GI tract, the child most likely would have diarrhea, not constipation. In general, chemotherapy should have no effect on skin integrity.

Which of the following nursing diagnoses would be inappropriate for the infant with gastroesophageal reflux (GER)? 1. Deficient fluid volume 2. Risk for aspiration 3. Imbalanced nutrition: Less than body requirements 4. Impaired oral mucous membrane

4. Impaired oral mucous membrane GER is the backflow of gastric contents into the esophagus resulting from relaxation or incompetence of the lower esophageal (cardiac) sphincter. No alteration in the oral mucous membranes occurs with this disorder. Fluid volume deficit, risk for aspiration, and imbalanced nutrition are appropriate nursing diagnoses.

When teaching parents about known antecedent infections in acute glomerulonephritis, which of the following should the nurse cover? 1. Herpes simplex 2. Scabies 3. Varicella 4. Impetigo

4. Impetigo Impetigo, a bacterial infection of the skin, may be caused by streptococci and may precede acute glomerulonephritis. Although most streptococcal infections do not cause acute glomerulonephritis, when they do, a latent period of 10 to 14 days occurs between the infection, usually of the skin (impetigo) or upper respiratory tract, and the onset of clinical manifestations. Herpes, scabies, and varicella are not associated with acute glomerulonephritis.

When caring for a child with increased intracranial pressure (ICP), which of the following if stated by the parents would indicate a need to reexplain the purpose for elevating the head of the bed at a 10- to 20-degree angle? 1. Maintain a neutral position. 2. Help alleviate headache. 3. Reduce intra-abdominal pressure. 4. Increase intrathoracic pressure.

4. Increase intrathoracic pressure. Head elevation decreases, not increases, intrathoracic pressure. Elevating the head of the bed in a child with increased ICP helps to maintain neutral position, alleviate headache, and reduce intra- abdominal pressure, which may contribute to increased ICP.

The nurse is planning care for a child with acute bacterial meningitis. Based on the mode of transmission of this infection, which precautionary intervention should be included in the plan of care? 1. Maintain enteric precautions. 2. Maintain neutropenic precautions. 3. No precautions are required as long as antibiotics have been started. 4. Maintain respiratory isolation precautions for at least 24 hours after the initiation of antibiotics.

4. Maintain respiratory isolation precautions for at least 24 hours after the initiation of antibiotics. Meningitis is an infectious process of the central nervous system caused by bacteria and viruses; it may be acquired as a primary disease or as a result of complications of neurosurgery, trauma, infection of the sinus or ears, or systemic infections. A major priority of nursing care for a child suspected to have meningitis is to administer the antibiotic as soon as it is prescribed. The child also is placed on respiratory isolation precautions for at least 24 hours while culture results are obtained and the antibiotic is having an effect. Enteric precautions and neutropenic precautions are not associated with the mode of transmission of meningitis. Enteric precautions are instituted when the mode of transmission is through the gastrointestinal tract. Neutropenic precautions are instituted when a child has a low neutrophil count.

A child with type 1 diabetes mellitus is brought to the emergency department by the mother, who states that the child has been complaining of abdominal pain and has been lethargic. Diabetic ketoacidosis is diagnosed. Anticipating the plan of care, the nurse prepares to administer which type of intravenous (IV) infusion? 1. Potassium infusion 2. NPH insulin infusion 3. 5% dextrose infusion 4. Normal saline infusion

4. Normal saline infusion Rationale: Diabetic ketoacidosis is a complication of diabetes mellitus that develops when a severe insulin deficiency occurs. Hyperglycemia occurs with diabetic ketoacidosis. Rehydration is the initial step in resolving diabetic ketoacidosis. Normal saline is the initial IV rehydration fluid. NPH insulin is never administered by the IV route. Dextrose solutions are added to the treatment when the blood glucose level decreases to an acceptable level. Intravenously administered potassium may be required, depending on the potassium level, but would not be part of the initial treatment. Cognitive Ability: Analyzing Client Needs: Physiological Integrity Integrated Process: Nursing Process: Planning Content Area: Pediatrics: Metabolic/Endocrine Strategy(ies): Subject Priority Concepts: Clinical Judgment, Glucose Regulation

The nurse analyzes the laboratory results of a child with hemophilia. The nurse understands that which result will most likely be abnormal in this child? 1. Platelet count 2. Hematocrit level 3. Hemoglobin level 4. Partial thromboplastin time

4. Partial thromboplastin time Hemophilia refers to a group of bleeding disorders resulting from a deficiency of specific coagulation proteins. Results of tests that measure platelet function are normal; results of tests that measure clotting factor function may be abnormal. Abnormal laboratory results in hemophilia indicate a prolonged partial thromboplastin time. The platelet count, hemoglobin level, and hematocrit level are normal in hemophilia. Cognitive Ability: Analyzing Client Needs: Physiological Integrity Integrated Process: Nursing Process: Assessment Content Area: Pediatrics: Hematological Strategy(ies): Strategic Words, Subject Priority Concepts: Clinical Judgment, Clotting

A 7-year-old child is seen in a clinic, and the health care provider documents a diagnosis of primary nocturnal enuresis. The nurse should provide which information to the parents? 1. Primary nocturnal enuresis does not respond to treatment. 2. Primary nocturnal enuresis is caused by a psychiatric problem. 3. Primary nocturnal enuresis requires surgical intervention to improve the problem. 4. Primary nocturnal enuresis is usually outgrown without therapeutic intervention.

4. Primary nocturnal enuresis is usually outgrown without therapeutic intervention. Primary nocturnal enuresis occurs in a child who has never been dry at night for extended periods. The condition is common in children, and most children eventually outgrow bed-wetting without therapeutic intervention. The child is unable to sense a full bladder and does not awaken to void. The child may have delayed maturation of the central nervous system. The condition is not caused by a psychiatric problem.

The nurse is caring for a child with hemophilia and is reviewing the results that were sent from the laboratory. Which result should the nurse expect in this child? 1. Shortened prothrombin time (PT) 2. Prolonged PT 3. Shortened partial thromboplastin time (PTT) 4. Prolonged PTT

4. Prolonged PTT PTT measures the activity of thromboplastin, which is dependent on intrinsic factors. In hemophilia, the intrinsic clotting factor VIII (antihemophilic factor) is deficient, resulting in a prolonged PTT. The results in the remaining options are incorrect. The PT may not necessarily be affected in this disorder. Cognitive Ability: Analyzing Client Needs: Physiological Integrity Integrated Process: Nursing Process: Assessment Content Area: Pediatrics: Hematological Strategy(ies): Subject Priority Concepts: Clinical Judgment, Clotting

Long-term complications of childhood cancer include which of the following? (Select all that apply.) 1. Strabismus 2. Chronic pancreatitis 3. Hypopituitarism 4. Pulmonary fibrosis 5. Gonad dysfunction 6. Second malignancies

4. Pulmonary fibrosis 5. Gonad dysfunction 6. Second malignancies Long-term effects of childhood cancer include pulmonary fibrosis, ovarian or testicular dysfunction, and second malignancies. Cataracts, chronic cirrhosis, and hypothyroidism are also possible.

A child in whom sickle cell anemia is suspected is seen in a clinic, and laboratory studies are performed. The nurse checks the laboratory results, knowing that which value would be increased in this disease? 1. Platelet count 2. Hematocrit level 3. Hemoglobin level 4. Reticulocyte count

4. Reticulocyte count A diagnosis is established on the basis of a complete blood count, examination for sickled red blood cells in the peripheral smear, and hemoglobin electrophoresis. Laboratory studies will show decreased hemoglobin level and hematocrit, a decreased platelet count, an increased reticulocyte count, and the presence of nucleated red blood cells. Reticulocyte counts are increased in children with sickle cell disease because the life span of their sickled red blood cells is shortened.

The nurse creates a plan of care for a child at risk for tonic-clonic seizures. In the plan of care, the nurse identifies seizure precautions and documents that which item(s) need to be placed at the child's bedside? 1. Emergency cart 2. Tracheotomy set 3. Padded tongue blade 4. Suctioning equipment and oxygen

4. Suctioning equipment and oxygen A seizure results from the excessive and unorganized neuronal discharges in the brain that activate associated motor and sensory organs. A type of generalized seizure is a tonic-clonic seizure. This type of seizure causes rigidity of all body muscles, followed by intense jerking movements. Because increased oral secretions and apnea can occur during and after the seizure, oxygen and suctioning equipment are placed at the bedside. A tracheotomy is not performed during a seizure. No object, including a padded tongue blade, is placed into the child's mouth during a seizure. An emergency cart would not be left at the bedside, but would be available in the treatment room or nearby on the nursing unit.

A diagnosis of Hodgkin's disease is suspected in a 12-year-old child. Several diagnostic studies are performed to determine the presence of this disease. Which diagnostic test result will confirm the diagnosis of Hodgkin's disease? 1. Elevated vanillylmandelic acid urinary levels 2. The presence of blast cells in the bone marrow 3. The presence of Epstein-Barr virus in the blood 4. The presence of Reed-Sternberg cells in the lymph nodes

4. The presence of Reed-Sternberg cells in the lymph nodes Hodgkin's disease (a type of lymphoma) is a malignancy of the lymph nodes. The presence of giant, multinucleated cells (Reed-Sternberg cells) is the classic characteristic of this disease. Elevated levels of vanillylmandelic acid in the urine may be found in children with neuroblastoma. The presence of blast cells in the bone marrow indicates leukemia. Epstein-Barr virus is associated with infectious mononucleosis.

The nurse provides feeding instructions to a parent of an infant diagnosed with gastroesophageal reflux disease. Which instruction should the nurse give to the parent to assist in reducing the episodes of emesis? 1. Provide less frequent, larger feedings. 2. Burp the infant less frequently during feedings. 3. Thin the feedings by adding water to the formula. 4. Thicken the feedings by adding rice cereal to the formula.

4. Thicken the feedings by adding rice cereal to the formula. Gastroesophageal reflux is backflow of gastric contents into the esophagus as a result of relaxation or incompetence of the lower esophageal or cardiac sphincter. Small, more frequent feedings with frequent burping often are prescribed in the treatment of gastroesophageal reflux. Feedings thickened with rice cereal may reduce episodes of emesis. If thickened formula is used, cross-cutting of the nipple may be required.

The primary reason for surgical repair of a myelomeningocele is which of the following? 1. To correct the neurologic defect 2. To prevent hydrocephalus 3. To prevent seizure disorders 4. To decrease the risk of infection

4. To decrease the risk of infection Surgical closure decreases the risk of infection stemming from damage to the fragile sac, which can lead to meningitis. The neurologic deficit cannot be corrected. However, some surgeons believe that early surgery reduces the risk of stretching spinal nerves and preventing further damage. Surgical repair does not help relieve hydrocephalus. In fact, some researchers believe that repair exaggerates the Arnold- Chiari malformation and decreases the absorptive surface for cerebrospinal fluid, leading to more rapid development of hydrocephalus. Surgical repair of the sac doesn't prevent seizure disorder, an impairment of the brain neuron tissue.

When assessing a child with encopresis, the interview is most likely to reveal a history of: 1. psychological disorder 2. sexual abuse 3. recurrent diarrhea 4. painful bowel movements

4. painful bowel movements In most cases of encopresis, children have a history of painful bowel movements or constipation. There is no good evidence to suggest that a psychological disorder, and most children with encopresis have not been sexually abused.

What is Hirschsprung's disease (aganglionic megacolon)?

A congenital anomaly results in mechanical obstruction b/c lack of ganglion cells don't allow food to move through intestines Affects the sigmoid colon

TARRY GREAN STOOLS SHOWS

ADQEAUTE IRON

The nurse is conducting staff in-service training on von Willebrand's disease. Which should the nurse include as characteristics of von Willebrand's disease? Select all that apply. 1. Easy bruising occurs. 2. Gum bleeding occurs. 3. It is a hereditary bleeding disorder. 4. Treatment and care are similar to that for hemophilia. 5. It is characterized by extremely high creatinine levels. 6. The disorder causes platelets to adhere to damaged endothelium.

ANS 1,2,3,4,6 von Willebrand's disease is a hereditary bleeding disorder characterized by a deficiency of or a defect in a protein termed von Willebrand factor. The disorder causes platelets to adhere to damaged endothelium. It is characterized by an increased tendency to bleed from mucous membranes. Assessment findings include epistaxis, gum bleeding, easy bruising, and excessive menstrual bleeding. An elevated creatinine level is not associated with this disorder. Cognitive Ability: Analyzing Client Needs: Physiological Integrity Integrated Process: Nursing Process: Assessment Content Area: Pediatrics: Hematological Strategy(ies): Subject Priority Concepts: Clinical Judgment, Clotting

A 12-year-old boy with a history of sickle cell anemia and a diagnosis of vaso-occlusive crisis is being assessed by the admitting nurse in the emergency department. Which of the following signs/symptoms would the nurse expect to see? Select all that apply. 1. Priapism 2. Pain level of 2/10 3. Hematuria 4. Elevated liver enzymes 5. Hematocrit 39%

ANS 1,3,4 Feedback 1: Priapism is symptom seen in males during a vaso-occlusive crisis. Feedback 2: The pain level is much higher during a vaso-occlusive crisis, often rated at 9/10 or 10/10 on a numeric pain rating scale. Feedback 3: Hematuria is a symptom seen during a vaso-occlusive crisis. Feedback 4: Elevated liver enzymes are seen during a vaso-occlusive crisis. Feedback 5: The nurse would expect to see a low hematocrit in a child with SCA. To remember signs and symptoms seen during vaso-occlusive crises, the test taker should remember the pathology of the attack (i.e., sickling and clumping of RBCs). Vascular organs, therefore, are most affected by the crisis. The blood becomes trapped in the vessels of the penis, resulting in a painful erection. The kidneys become ischemic, resulting in the loss of blood into the urine. The liver becomes ischemic, resulting in elevated liver enzymes. Patients in vaso-occlusive crisis experience severe pain.

An 18-month-old male is brought to the clinic by his mother. His height is in the 50th percentile, and weight is in the 80th percentile. The child is pale. The physical examination is normal, but his hematocrit level is 20%. Which of the following questions should assist the nurse in making a diagnosis? Select all that apply. 1. "How many bowel movements a day does your child have?" 2. "How much did your baby weigh at birth?" 3. "What does your child eat every day?" 4. "Has the child been given any new medications?" 5. "How much milk does your child drink per day?"

ANS 3, 5. 1. Because the child has a low hematocrit level, the child most likely has anemia. Anemia in a child is usually of nutritional origin. Iron-deficiency anemia is the most common nutritional anemia. This is important information but not necessary to make the diagnosis of iron-deficiency anemia. 2. Knowing birth weight can help determine if the child is following his or her own curve on the growth chart. 3. A diet history is necessary to determine the nutritional status of the child and if the child is getting sufficient sources of iron. 4. Knowing if the child is taking any new medication is good but is not necessary to make the diagnosis of iron-deficiency anemia. 5. By asking how much milk the child consumes, the nurse can determine if the child is filling up on milk and then not wanting to take food. TEST-TAKING HINT: The most common anemia in children and in toddlers is iron deficiency anemia, frequently due to drinking too much milk and not eating enough iron-rich foods

Nursing care of the child with Kawasaki disease is challenging because of: a. the child's irritability. b. predictable disease course. c. complex antibiotic therapy. d. the child's ongoing requests for food.

ANS: A Patient irritability is a hallmark of Kawasaki disease and the most challenging problem. A quiet environment is necessary to promote rest. The diagnosis is often difficult to make, and the course of the disease can be unpredictable. Intravenous gamma globulin and salicylates are the therapy of choice, not antibiotics. The child often is reluctant to eat. Soft foods and fluids should be offered to prevent dehydration.

Which of the following defects results in obstruction to blood flow? a. Aortic stenosis b. Tricuspid atresia c. Atrial septal defect d. Transposition of the great arteries

ANS: A Aortic stenosis is a narrowing or stricture of the aortic valve, causing resistance to blood flow in the left ventricle, decreased cardiac output, left ventricular hypertrophy, and pulmonary vascular congestion. Tricuspid atresia results in decreased pulmonary blood flow. The atrial septal defect results in increased pulmonary blood flow. Transposition of the great arteries results in mixed blood flow.

Seventy-two hours after cardiac surgery, a young child has a temperature of 38.4o C (101.1° F). The nurse should do which of the following? a. Report findings to practitioner. b. Apply a hypothermia blanket. c. Keep child warm with blankets. d. Record temperature on assessment flow sheet.

ANS: A In the first 24 to 48 hours after surgery, the body temperature may increase to 37.8° C (100° F) as part of the inflammatory response to tissue trauma. If the temperature is higher or fever continues after this period, it is most likely a sign of an infection, and immediate investigation is indicated. Hypothermia blanket is not indicated for this level of temperature. Blankets should be removed from the child to keep the temperature from increasing. The temperature should be recorded, but the practitioner must be notified for evaluation.

Which of the following structural defects constitute tetralogy of Fallot? a. Pulmonary stenosis, ventricular septal defect, overriding aorta, right ventricular hypertrophy b. Aortic stenosis, ventricular septal defect, overriding aorta, right ventricular hypertrophy c. Aortic stenosis, ventricular septal defect, overriding aorta, left ventricular hypertrophy d. Pulmonary stenosis, ventricular septal defect, aortic hypertrophy, left ventricular hypertrophy

ANS: A Tetralogy of Fallot has these four characteristics: pulmonary stenosis, ventricular septal defect, overriding aorta, and right ventricular hypertrophy.

The primary nursing intervention to prevent bacterial endocarditis is which of the following? a. Counsel parents of high-risk children. b. Institute measures to prevent dental procedures. c. Encourage restricted mobility in susceptible children. d. Observe children for complications, such as embolism and heart failure.

ANS: A The objective of nursing care is to counsel the parents of high-risk children about the need for both prophylactic antibiotics for dental procedures and maintaining excellent oral health. The child's dentist should be aware of the child's cardiac condition. Dental procedures should be done to maintain a high level of oral health. Restricted mobility in susceptible children is not indicated. Parents are taught to observe for unexplained fever, weight loss, or change in behavior.

A parent of a 7-year-old girl with a repaired ventricular septal defect (VSD) calls the cardiology clinic and reports that the child is just not herself. Her appetite is decreased, she has had intermittent fevers around 38o C (100.4o F), and now her muscles and joints ache. Based on this information you advise the mother to: a. immediately bring the child to clinic for evaluation. b. come to the clinic next week on a scheduled appointment. c. treat the symptoms with acetaminophen and fluids, since it is most likely a viral illness. d. recognize that the child is trying to manipulate the parent by complaining of vague symptoms.

ANS: A These are the insidious symptoms of bacterial endocarditis. Since the child is in a high-risk group for this disorder (VSD repair), immediate evaluation and treatment are indicated to prevent cardiac damage. With appropriate antibiotic therapy, bacterial endocarditis is successfully treated in approximately 80% of the cases. The child's complaints should not be dismissed. The low-grade fever is not a symptom that the child can fabricate.

A school-age child had an upper respiratory tract infection for several days and then began having a persistent dry, hacking cough that was worse at night. The cough has become productive in the past 24 hours. This is most suggestive of a. Bronchitis b. Bronchiolitis c. Viral-induced asthma d. Acute spasmodic laryngitis

ANS: A Feedback A Bronchitis is characterized by these symptoms and occurs in children older than 6 years. B Bronchiolitis is rare in children older than 2 years. C Asthma is a chronic inflammation of the airways that may be exacerbated by a virus. D Acute spasmodic laryngitis occurs in children between 3 months and 3 years.

A child has a chronic, nonproductive cough and diffuse wheezing during the expiratory phase of respiration. This suggests a. Asthma b. Pneumonia c. Bronchiolitis d. Foreign body in trachea

ANS: A Feedback A Children with asthma usually have these chronic symptoms. B Pneumonia appears with an acute onset and fever and general malaise. C Bronchiolitis is an acute condition caused by RSV. D Foreign body in the trachea will occur with an acute respiratory distress or failure and maybe stridor.

An infant's parents ask the nurse about preventing OM. What should be recommended? a. Avoid tobacco smoke. b. Use nasal decongestant. c. Avoid children with OM. d. Bottle feed or breastfeed in supine position.

ANS: A Feedback A Eliminating tobacco smoke from the child's environment is essential for preventing OM and other common childhood illnesses. B Nasal decongestants are not useful in preventing OM. C Children with uncomplicated OM are not contagious unless they show other upper respiratory infection (URI) symptoms. D Children should be fed in an upright position to prevent OM.

The nurse should assess a child who has had a tonsillectomy for a. Frequent swallowing b. Inspiratory stridor c. Rhonchi d. Elevated white blood cell count

ANS: A Feedback A Frequent swallowing is indicative of postoperative bleeding. B Inspiratory stridor is characteristic of croup. C Rhonchi are lower airway sounds indicating pneumonia. D Assessment of blood cell counts is part of a preoperative workup.

What sign is indicative of respiratory distress in infants? a. Nasal flaring b. Respiratory rate of 55 breaths/min c. Irregular respiratory pattern d. Abdominal breathing

ANS: A Feedback A Infants have difficulty breathing through their mouths; therefore nasal flaring is usually accompanied by extra respiratory efforts. It also allows more air to enter as the nares flare. B A respiratory rate of 55 breaths/min is a normal assessment for an infant. Tachypnea is a respiratory rate of 60 to 80 breaths/min. C Irregular respirations are normal in the infant. D Abdominal breathing is common because the diaphragm is the neonate's major breathing muscle.

The nurse expects the initial plan of care for a 9-month-old child with an acute otitis media infection to include a. symptomatic treatment and observation for 48 to 72 hours after diagnosis b. an oral antibiotic, such as amoxicillin, five times a day for 7 days c. pneumococcal conjugate vaccine d. myringotomy with tympanoplasty tubes

ANS: A Feedback A Select children 6 months of age or older with acute otitis media are treated by initiating symptomatic treatment and observation for 48 to 72 hours. B Acute otitis media may be treated with a 5- to 10-day course of oral antibiotics. When treatment is indicated, amoxicillin at a divided dose of 80 to 90 mg/kg/day given either every 8 or 12 hours for 5 to 10 days may be ordered. C Pneumococcal conjugate vaccine helps to prevent ear infections but is not included in the initial plan of care for a child with acute otitis media. D Surgical intervention is considered when the child has persistent ear infection despite antibiotic therapy or with otitis media with effusion that persists for more than 3 months and is associated with hearing loss.

Which classification of drugs is used to relieve an acute asthma episode? a. Short-acting beta2-adrenergic agonist b. Inhaled corticosteroids c. Leukotriene blockers d. Long-acting bronchodilators

ANS: A Feedback A Short-acting beta2-adrenergic agonist is the first medication administered. Later, systemic corticosteroids decrease airway inflammation in an acute asthma attack. They are given for short courses of 5 to 7 days. B Inhaled corticosteroids are used for long-term, routine control of asthma. C Leukotriene blockers diminish the mediator action of leukotrienes and are used for long-term, routine control of asthma in children older than 12 years. D A long-acting bronchodilator would not relieve acute symptoms.

The father of an infant calls the nurse to his son's room because he is "making a strange noise." A diagnosis of laryngomalacia is made. What does the nurse expect to find on assessment? a. Stridor b. High-pitched cry c. Nasal congestion d. Spasmodic cough

ANS: A Feedback A Stridor is usually present at birth but may begin as late as 2 months. Symptoms increase when the infant is supine or crying. B High-pitched cries are consistent with neurologic abnormalities and are not usually respiratory in nature. C Nasal congestion is nonspecific in relation to laryngomalacia. D Spasmodic cough is associated with croup; it is not a common symptom of laryngomalacia.

The nurse getting an end-of-shift report on a child with status asthmaticus should question which intervention? a. Administer oxygen by nasal cannula to keep oxygen saturation at 100%. b. Assess intravenous (IV) maintenance fluids and site every hour. c. Notify physician for signs of increasing respiratory distress. d. Organize care to allow for uninterrupted rest periods.

ANS: A Feedback A Supplemental oxygen should not be administered to maintain oxygen saturation at 100%. Keeping the saturation around 95% is adequate. Administration of too much oxygen to a child may lead to respiratory depression by decreasing the stimulus to breathe, leading to carbon dioxide retention. B When the child cannot take oral fluids because of respiratory distress, IV fluids are administered. The child with a continuous IV infusion must be assessed hourly to prevent complications. C A physician should be notified of any changes indicating increasing respiratory distress. D A child in respiratory distress is easily fatigued. Nursing care should be organized so the child can get needed rest without being disturbed.

What is the earliest recognizable clinical manifestation(s) of CF? a. Meconium ileus b. History of poor intestinal absorption c. Foul-smelling, frothy, greasy stools d. Recurrent pneumonia and lung infections

ANS: A Feedback A The earliest clinical manifestation of CF is a meconium ileus, which is found in about 10% of children with CF. Clinical manifestations include abdominal distention, vomiting, failure to pass stools, and rapid development of dehydration. B History of malabsorption is a later sign that manifests as failure to thrive. C Foul-smelling stools are a later manifestation of CF. D Recurrent respiratory infections are a later sign of CF.

What explanation should the nurse give to the parent of a child with asthma about using a peak flow meter? a. It is used to monitor the child's breathing capacity. b. It measures the child's lung volume. c. It will help the medication reach the child's airways. d. It measures the amount of air the child breathes in.

ANS: A Feedback A The peak flow meter is a device used to monitor breathing capacity in the child with asthma. B A child with asthma would have a pulmonary function test to measure lung volume. C A spacer used with a metered-dose inhaler prolongs medication transit so medication reaches the airways. D The peak flow meter measures the flow of air in a forced exhalation in liters per minute.

A 5-year-old child is brought to the emergency department with copious drooling and a croaking sound on inspiration. Her mother states that the child is very agitated and only wants to sit upright. What should be the nurse's first action in this situation? a. Prepare intubation equipment and call the physician. b. Examine the child's oropharynx and call the physician. c. Obtain a throat culture for respiratory syncytial virus (RSV). d. Obtain vital signs and listen to breath sounds.

ANS: A Feedback A This child has symptoms of epiglottitis, is acutely ill, and requires emergency measures. B If epiglottitis is suspected, the nurse should not examine the child's throat. Inspection of the epiglottis is only done by a physician, because it could trigger airway obstruction. C A throat culture could precipitate a complete respiratory obstruction. D Vital signs can be assessed after emergency equipment is readied.

Which assessment finding after tonsillectomy should be reported to the physician? a. Vomiting bright red blood b. Pain at surgical site c. Pain on swallowing d. The ability to only take small sips of liquids

ANS: A Feedback A Vomiting bright red blood and swallowing frequently are signs of bleeding postoperatively and should be reported to the physician. B It is normal for the child to have pain at the surgical site. C It is normal for the child to have pain on swallowing. D Only clear liquids are offered immediately after surgery, and small sips are preferred.

The nurse should teach parents of a child with cystic fibrosis to adjust enzyme dosage according to which indicator? a. Stool formation b. Vomiting c. Weight d. Urine output

ANS: A Feedback A When there is constipation, less enzyme is needed; with steatorrhea, more enzyme is needed for digestion of nutrients. B Vomiting does not affect enzyme dosaging. C The child's weight does not affect enzyme dosaging. D Urine output is not relevant to enzyme replacement.

What information should the nurse teach families about reducing exposure to pollens and dust? Select all that apply. a. Replace wall-to-wall carpeting with wood and tile floors. b. Use an air conditioner. c. Put dust-proof covers on pillows and mattresses. d. Keep humidity in the house above 60%. e. Keep pets outside.

ANS: A, B, C Feedback Correct: Carpets retain dust. To reduce exposure to dust, carpeting should be replaced with wood, tile, slate, or vinyl. These floors can be cleaned easily. For anyone with pollen allergies, it is best to keep the windows closed and to run the air conditioner. Covering mattresses and pillows with dust-proof covers will reduce exposure to dust. Incorrect: A humidity level above 60% promotes dust mites. It is recommended that household humidity be kept between 40% and 50% to reduce dust mites inside the house. Keeping pets outside will help to decrease exposure to dander, but will not affect exposure to pollen and dust.

A nurse is planning care for an asymptomatic child with a positive tuberculin test. What should the nurse include in the plan? Select all that apply. a. Administration of daily isoniazid (INH) b. Instructing family members about administration of INH to all close contacts of the child c. Administration of the Bacillus Calmette-Guérin vaccine d. Reporting the case to the health department e. Administration of INH and rifampin (Rifadin) simultaneously

ANS: A, B, D Feedback Correct: After a chest radiograph is obtained, asymptomatic children with positive tuberculin tests and no previous history of TB receive daily INH for 9 months. Asymptomatic contacts should receive INH for at least 8 to 10 weeks after contact has been broken or until a negative skin test can be confirmed (a second test is taken at least 10 weeks after the last exposure). Reporting cases of TB is required by law in all states in the United States. Incorrect: Bacillus Calmette-Guérin vaccine is the only anti-TB vaccine available, but it is given only to children who have negative test results. For asymptomatic TB, only INH is administered, not both isoniazid and rifampin together. Rifampin is used if the child has resistance to isoniazid.

The mother of a newborn asks the nurse what causes the baby to begin to breathe after delivery. What changes in the respiratory system stimulating respirations postnatally can the nurse explain to the mother? Select all that apply. a. Low oxygen levels in the infant's blood b. Rubbing the newborn with a towel or blanket c. Surfactant, a special lubricant in the lungs d. Increased blood flow to the infant's lungs e. Cold environment in the delivery room

ANS: A, B, E Feedback Correct: A postnatal change in the respiratory system is the stimulation of respiration by hypoxemia, hypercarbia, cold, tactile stimulation, and a possible decrease in the concentration of prostaglandin E2. Incorrect: Surfactant in the lungs lowers surface tension and facilitates lung expansion. It does not stimulate respirations. Pulmonary blood flow increases after birth, but this does not stimulate respirations in the newborn.

The nurse should implement which interventions for an infant experiencing apnea? Select all that apply. a. Stimulate the infant by gently tapping the foot. b. Shake the infant vigorously. c. Have resuscitative equipment available. d. Suction the infant. e. Maintain a neutral thermal environment.

ANS: A, C, E Feedback Correct: An infant with apnea should be stimulated by gently tapping the foot. Resuscitative equipment should be available and the infant should be maintained in a neutral thermal environment. Incorrect: The infant should not be shaken vigorously nor suctioned.

A cardiac defect that allows blood to shunt from the (high pressure) left side of the heart to the (lower pressure) right side can result in: a. cyanosis. b. congestive heart failure. c. decreased pulmonary blood flow. d. bounding pulses in upper extremities.

ANS: B As blood is shunted into the right side of the heart, there is increased pulmonary blood flow and the child is at high risk for congestive heart failure. Cyanosis usually occurs in defects with decreased pulmonary blood flow. Bounding upper extremity pulses are a manifestation of coarctation of the aorta.

The nurse should recognize that congestive heart failure (CHF) is which of the following? a. Disease related to cardiac defects b. Consequence of an underlying cardiac defect c. Inherited disorder associated with a variety of defects d. Result of diminished workload imposed on an abnormal myocardium

ANS: B CHF is the inability of the heart to pump an adequate amount of blood to the systemic circulation at normal filling pressures to meet the body's metabolic demands. CHF is not a disease but rather a result of the inability of the heart to pump efficiently. CHF is not inherited. CHF occurs most frequently secondary to congenital heart defects in which structural abnormalities result in increased volume load or increased pressures on the ventricles.

The nurse is caring for a child with persistent hypoxia secondary to a cardiac defect. The nurse recognizes the risk of cerebrovascular accidents (strokes) occurring. Which of the following is an important objective to decrease this risk? a. Minimize seizures. b. Prevent dehydration. c. Promote cardiac output. d. Reduce energy expenditure.

ANS: B In children with persistent hypoxia, polycythemia develops. Dehydration must be prevented in hypoxemic children because it potentiates the risk of strokes. Minimizing seizures, promoting cardiac output, and reducing energy expenditure will not reduce the risk of cerebrovascular accidents.

John is a 6-year-old child scheduled for a cardiac catheterization. Preoperative teaching should be which of the following? a. Directed at his parents because he is too young to understand b. Adapted to his level of development so that he can understand c. Done several days before the procedure so he will be prepared d. Provide details about the actual procedures so he will know what to expect

ANS: B Preoperative teaching should always be directed to the child's stage of development. The caregivers also benefit from these explanations. The parents may ask additional questions, which should be answered, but the child needs to receive the information based on developmental level. This age-group will not understand in-depth descriptions. School-age children should be prepared close to the time of the cardiac catheterization.

Which of the following should the nurse consider when preparing a school-age child and the family for heart surgery? a. Unfamiliar equipment should not be shown. b. Let child hear the sounds of a cardiac monitor, including alarms. c. Explain that an endotracheal tube will not be needed if the surgery goes well. d. Discussion of postoperative discomfort and interventions is not necessary before the procedure.

ANS: B The child and family should be exposed to the sights and sounds of the intensive care unit (ICU). All positive, nonfrightening aspects of the environment are emphasized. The family and child should make the decision about a tour of the unit if it is an option. The child should be shown unfamiliar equipment and its use demonstrated on a doll. Carefully prepare the child for the postoperative experience, including intravenous lines, incision, endotracheal tube, expected discomfort, and management strategies.

The parents of a 3-year-old child with congenital heart disease are afraid to let their child play with other children because of possible overexertion. The nurse's reply should include which of the following? a. Parents can meet all the child's needs. b. Child needs opportunities to play with peers. c. Constant parental supervision is needed to avoid overexertion. d. Child needs to understand that peers' activities are too strenuous.

ANS: B The child needs opportunities for social development. Children are able to regulate and limit their activities based on their energy level. Parents must be encouraged to seek appropriate social activities for the child, especially before kindergarten. The child needs to have activities that foster independence.

Ventricular septal defect has the following blood flow pattern: a. Mixed blood flow b. Increased pulmonary blood flow c. Decreased pulmonary blood flow d. Obstruction to blood flow from ventricles

ANS: B The opening in the septal wall allows for blood to flow from the higher pressure left ventricle into the lower pressure right ventricle. This left-to-right shunt creates increased pulmonary blood flow. The shunt is one way, from high pressure to lower pressure; oxygenated and unoxygenated blood do not mix. The outflow of blood from the ventricles is not affected by the septal defect.

The nurse notices that a child is increasingly apprehensive and has tachycardia after heart surgery. The chest tube drainage is now 8 ml/kg/hr. Which of the following should be the nurse's initial intervention? a. Apply warming blankets. b. Notify the practitioner of these findings. c. Give additional pain medication per protocol. d. Encourage child to cough, turn, and deep breathe.

ANS: B The practitioner is notified immediately. Increases of chest tube drainage to more than 3 ml/kg/hr for more than 3 consecutive hours or 5 to 10 ml/kg in any 1 hour may indicate postoperative hemorrhage. Increased chest tube drainage with apprehensiveness and tachycardia may indicate cardiac tamponade—blood or fluid in the pericardial space constricting the heart—which is a life-threatening complication. Warming blankets are not indicated at this time. Additional pain medication can be given before the practitioner drains the fluid, but the notification is the first action. Encouraging the child to cough, turn, and deep breathe should be deferred until after evaluation by the practitioner.

What is the best nursing response to the parent of a child with asthma who asks if his child can still participate in sports? a. "Children with asthma are usually restricted from physical activities." b. "Children can usually play any type of sport if their asthma is well controlled." c. "Avoid swimming because breathing underwater is dangerous for people with asthma." d. "Even with good asthma control, I would advise limiting the child to one athletic activity per school year."

ANS: B Feedback A Children with asthma should not be restricted from physical activity. B Sports that do not require sustained exertion, such as gymnastics, baseball, and weight lifting, are well tolerated. Children can usually play any type of sport if their asthma is well controlled. C Swimming is recommended as the ideal sport for children with asthma because the air is humidified and exhaling underwater prolongs exhalation and increases end-expiratory pressure. D If asthma is well controlled, the child can participate in any type of sport.

Which statement indicates that a parent of a toddler needs more education about preventing foreign body aspiration? a. "I keep objects with small parts out of reach." b. "My toddler loves to play with balloons." c. "I won't permit my child to have peanuts." d. "I never leave coins where my child could get them."

ANS: B Feedback A Keeping toys with small parts and other small objects out of reach can prevent foreign body aspiration. B Latex balloons account for a significant number of deaths from aspiration every year. C Peanuts are just one of the foods that pose a choking risk if given to young children. D Small objects, such as coins, need to be put out of the small child's reach.

Which type of croup is always considered a medical emergency? a. Laryngitis b. Epiglottitis c. Spasmodic croup d. Laryngotracheobronchitis (LTB)

ANS: B Feedback A Laryngitis is a common viral illness in older children and adolescents, with hoarseness and URI symptoms. B Epiglottitis is always a medical emergency that requires antibiotics and airway support for treatment. C Spasmodic croup is treated with humidity. D LTB may progress to a medical emergency in some children.

The nurse is caring for a child with acute respiratory distress syndrome (ARDS) associated with sepsis. Nursing actions should include: a. Forcing fluids b. Monitoring pulse oximetry c. Instituting seizure precautions d. Encouraging a high-protein diet

ANS: B Feedback A Maintenance of vascular volume and hydration is important and should be done parenterally. B Monitoring cardiopulmonary status is an important evaluation tool in the care of the child with ARDS. C Seizures are not a side effect of ARDS. D Adequate nutrition is necessary, but a high-protein diet is not helpful.

The infant with bronchopulmonary dysplasia (BPD) who has RSV bronchiolitis is a candidate for which treatment? a. Pancreatic enzymes b. Cool humidified oxygen c. Erythromycin intravenously d. Intermittent positive pressure ventilation

ANS: B Feedback A Pancreatic enzymes are used for patients with cystic fibrosis. B Humidified oxygen is delivered if the oxygen saturation level drops to less than 90%. C Antibiotics are ineffective against viral illnesses. Oxygen can be administered by hood, facemask, or nasal cannula. D Assisted ventilation is not necessary in the treatment of RSV infections.

Which statement made by a parent indicates an understanding about the genetic transmission of cystic fibrosis (CF)? a. "Only one parent carries the cystic fibrosis gene." b. "Both parents are carriers of the cystic fibrosis gene." c. "The presence of the disease is most likely the result of a genetic mutation." d. "The mother is usually the carrier of the cystic fibrosis gene."

ANS: B Feedback A The disease will not be present if only one parent is a carrier of the cystic fibrosis gene. B Cystic fibrosis follows a pattern of autosomal recessive transmission. Both parents must be carriers of the gene for the disease to be transmitted to the child. If both parents carry the CF gene, each pregnancy has a 25% chance of producing a CF-affected child. C Cystic fibrosis is known to have a definite pattern of transmission. It is transmitted as an autosomal recessive trait. D A carrier parent can transmit the carrier gene to the child. The disease is present when the carrier gene is transmitted from both parents.

The nurse encourages the mother of a toddler with acute LTB to stay at the bedside as much as possible. The nurse's rationale for this action is primarily that a. Mothers of hospitalized toddlers often experience guilt. b. The mother's presence will reduce anxiety and ease child's respiratory efforts. c. Separation from mother is a major developmental threat at this age. d. The mother can provide constant observations of the child's respiratory efforts.

ANS: B Feedback A This is true, but not the best answer. B The family's presence will decrease the child's distress. C Although true for toddlers, the main reason to keep parents at the child's bedside is to ease anxiety and therefore respiratory effort. D The child should have constant monitoring by cardiorespiratory monitor and noninvasive oxygen saturation monitoring, but the parent should not play this role in the hospital.

What intervention can be taught to the parents of a 3-year-old child with pneumonia who is not hospitalized? a. Offer the child only cool liquids. b. Offer the child her favorite warm liquid drinks. c. Use a warm mist humidifier. d. Call the physician for a respiratory rate less than 28 breaths/min.

ANS: B Feedback A Warm liquids are preferable because they help loosen secretions. B Offering the child fluids that she likes will facilitate oral intake. Warm liquids help loosen secretions. C Cool mist humidifiers are preferred to warm mist. Warm mist is a safety concern and could cause burns if touched by the child. D Typically parents are not taught to count their children's respirations and report abnormalities to the physician. Even if this were the case, a respiratory rate of less than 28 breaths/min is normal for a 3-year-old child. The expected respiratory rate for a 3-year-old child is 20 to 30 breaths/min.

The parent of a toddler calls the nurse, asking about croup. What is a distinguishing manifestation of spasmodic croup? a. Wheezing is heard audibly. b. It has a harsh, barky cough. c. It is bacterial in nature. d. The child has a high fever.

ANS: B Feedback A Wheezing is not a distinguishing manifestation of croup. It can accompany conditions such as asthma or bronchiolitis. B Spasmodic croup is viral in origin; is usually preceded by several days of symptoms of upper respiratory tract infection; often begins at night; and is marked by a harsh, metallic, barky cough; sore throat; inspiratory stridor; and hoarseness. C Spasmodic croup is viral in origin. D A high fever is not usually present.

SELECT ALL THAT APPLY. The nurse is caring for a child with Kawasaki disease in the acute phase. Which of the following clinical manifestations would the nurse expect to observe? a. Osler nodes b. Cervical lymphadenopathy c. Strawberry tongue d. Chorea e. Erythematous palms f. Polyarthritis

ANS: B, C, E Clinical manifestations of Kawasaki disease in the acute phase include cervical lymphadenopathy, a strawberry tongue, and erythematous palms. Osler nodes are a clinical manifestation of endocarditis. Chorea and polyarthritis are seen in rheumatic fever.

Decreasing the demands on the heart is a priority in care for the infant with congestive heart failure (CHF). In evaluating the infant's status, which of the following is indicative of achieving this goal? a. Irritability when awake b. Capillary refill of more than 5 seconds c. Appropriate weight gain for age d. Positioned in high Fowler position to maintain oxygen saturation at 90%

ANS: C Appropriate weight gain for an infant is indicative of successful feeding and a reduction in caloric loss secondary to the CHF. Irritability is a symptom of CHF. The child also uses additional energy when irritable. Capillary refill should be brisk and within 2 to 3 seconds. The child needs to be positioned upright to maintain oxygen saturation at 90%. Positioning is helping to decrease respiratory effort, but the infant is still having difficulty with oxygenation.

The nurse finds that a 6-month-old infant has an apical pulse of 166 beats/min during sleep. The nurse should do which of the following? a. Administer oxygen. b. Record data on nurses' notes. c. Report data to the practitioner. d. Place child in high Fowler position.

ANS: C One of the earliest signs of CHF is tachycardia (sleeping heart rate >160 beats/min) as a direct result of sympathetic stimulation. The practitioner needs to be notified for evaluation of possible CHF. Although oxygen or a semiupright position may be indicated, the first action is to report the data to the practitioner.

After returning from cardiac catheterization, the nurse determines that the pulse distal to the catheter insertion site is weaker. The nurse should do which of the following? a. Elevate affected extremity. b. Notify practitioner of the observation. c. Record data on assessment flow record. d. Apply warm compresses to insertion site.

ANS: C The pulse distal to the catheterization site may be weaker for the first few hours after catheterization, but should gradually increase in strength. Documentation of the finding provides a baseline. The extremity is maintained straight for 4 to 6 hours. This is an expected change. The pulse is monitored. If there are neurovascular changes in the extremity, the practitioner is notified. The site is kept dry. Warm compresses are not indicated.

Teaching safety precautions with the administration of antihistamines is important because of what common side effect? a. Dry mouth b. Excitability c. Drowsiness d. Dry mucous membranes

ANS: C Feedback A A dry mouth is not a safety issue. B Excitability may affect rest or sleep, but drowsiness is the most important safety hazard. C Drowsiness is a safety hazard when alertness is needed, especially with a teenage driver. Nonsedating brands should be used. D Dry mucous membranes are not a safety issue.

Which intervention for treating croup at home should be taught to parents? a. Have a decongestant available to give the child when an attack occurs. b. Have the child sleep in a dry room. c. Take the child outside. d. Give the child an antibiotic at bedtime.

ANS: C Feedback A Decongestants are inappropriate for croup, which affects the middle airway level. B A dry environment may contribute to symptoms. C Taking the child into the cool, humid, night air may relieve mucosal swelling and improve symptoms. D Croup is caused by a virus. Antibiotic treatment is not indicated.

What is a common trigger for asthma attacks in children? a. Febrile episodes b. Dehydration c. Exercise d. Seizures

ANS: C Feedback A Febrile episodes are consistent with other problems, for example, seizures. B Dehydration occurs as a result of diarrhea; it does not trigger asthma attacks. Viral infections are triggers for asthma. C Exercise is one of the most common triggers for asthma attacks, particularly in school-age children. D Seizures can result from a too-rapid intravenous infusion of theophylline—a therapy for asthma.

Which finding confirms a diagnosis of cystic fibrosis? a. Chest radiograph shows alveolar hyperinflation. b. Stool analysis indicates significant amounts of fecal fat. c. Sweat chloride is greater than 60 mEq/L. d. Liver function levels are abnormal.

ANS: C Feedback A Hyperinflation is one of the first findings on a chest radiograph of a child with cystic fibrosis. It does not confirm a diagnosis. B A 72-hour fecal fat determination may be included in a diagnostic workup. Inability to secrete digestive enzymes causes steatorrhea. C The diagnosis of cystic fibrosis requires a positive sweat test. A chloride level greater than 60 mEq/L is considered diagnostic for cystic fibrosis. D Liver function tests may be part of the diagnostic workup for cystic fibrosis.

A small child with cystic fibrosis cannot swallow pancreatic enzyme capsules. The nurse should teach parents to mix enzymes with: a. Macaroni and cheese b. Tapioca c. Applesauce d. Hot chocolate

ANS: C Feedback A Macaroni and cheese is not a good choice because enzymes are inactivated by heat and starchy foods. B Tapioca is not a good choice because enzymes are inactivated by starchy foods. C Enzymes can be mixed with a small amount of nonacidic foods. D Enzymes are less effective if mixed with foods that are hot.

Once an allergen is identified in a child with allergic rhinitis, the treatment of choice about which to educate the parents is a. Using appropriate medications b. Beginning desensitization injections c. Eliminating the allergen d. Removing the adenoids

ANS: C Feedback A Medications are not a first-line treatment but can be helpful in controlling allergic rhinitis. B Immunotherapy is usually the final component of controlling allergic rhinitis. C The first priority is to attempt to remove the causative agent from the child's environment. D Adenoids are tissues that can swell with constant rhinitis; however, a surgical procedure is not indicated for allergic rhinitis. Dealing with the cause is the first priority.

Which child requires a Mantoux test? a. The child who has episodes of nighttime wheezing and coughing b. The child who has a history of allergic rhinitis c. The child whose baby-sitter has received a tuberculosis diagnosis d. The premature infant who is being treated for apnea of infancy

ANS: C Feedback A Nighttime wheezing and coughing are consistent with a diagnosis of asthma. B Allergic rhinitis requires an allergy workup. C The Mantoux test is the initial screening mechanism for patients exposed to tuberculosis. D This infant requires a sleep study as part of the evaluation.

Which statement, if made by parents of a child with cystic fibrosis, indicates that they understood the nurse's teaching on pancreatic enzyme replacement? a. "Enzymes will improve my child's breathing." b. "I should give the enzymes 1 hour after meals." c. "Enzymes should be given with meals and snacks." d. "The enzymes are stopped if my child begins wheezing."

ANS: C Feedback A Pancreatic enzymes do not affect the respiratory system. B Pancreatic enzymes are taken within 30 minutes of eating all meals and snacks. Giving the medication 1 hour after meals is inappropriate and ineffective for absorption of nutrients. C Children with cystic fibrosis need to take enzymes with food for adequate absorption of nutrients. D Wheezing is not a reason to stop taking enzyme replacements.

Which statement made by a parent indicates an understanding about treatment of streptococcal pharyngitis? a. "I guess my child will need to have his tonsils removed." b. "A couple of days of rest and some ibuprofen will take care of this." c. "I should give the penicillin three times a day for 10 days." d. "I am giving my child prednisone to decrease the swelling of the tonsils."

ANS: C Feedback A Surgical removal of the tonsils is a controversial issue. It may be warranted in cases of recurrent tonsillitis. It is not indicated for the treatment of acute tonsillitis. B Comfort measures such as rest and analgesics are indicated, but these will not treat the bacterial infection. C Streptococcal pharyngitis is best treated with oral penicillin two to three times daily for 10 days. D Corticosteroids are not used in the treatment of streptococcal pharyngitis.

For which problem should the child with chronic otitis media with effusion be evaluated? a. Brain abscess b. Meningitis c. Hearing loss d. Perforation of the tympanic membrane

ANS: C Feedback A The infection of acute otitis media can spread to surrounding tissues, causing a brain abscess. B The infection of acute otitis media can spread to surrounding tissues, causing meningitis. C Chronic otitis media with effusion is the most common cause of hearing loss in children. D Inflammation and pressure from acute otitis media may result in perforation of the tympanic membrane.

What should the nurse teach a child about using an albuterol metered-dose inhaler for exercise-induced asthma? a. Take two puffs every 6 hours around the clock. b. Use the inhaler only when the child is short of breath. c. Use the inhaler 30 minutes before exercise. d. Take one to two puffs every morning upon awakening.

ANS: C Feedback A This schedule will not relieve exercise-induced asthma. B Waiting until symptoms are severe is too late to begin using a metered-dose inhaler. C The appropriate time to use an inhaled beta2-agonist or cromolyn is before an event that could trigger an attack. D This may be the child's usual schedule for medication. If exercise causes symptoms, additional medication is indicated.

When caring for the child with Kawasaki disease, the nurse should know which of the following? a. Aspirin is contraindicated. b. Principal area of involvement is the joints. c. Child's fever is usually responsive to antibiotics within 48 hours. d. Therapeutic management includes administration of gamma globulin and salicylates.

ANS: D High-dose intravenous gamma globulin and salicylate therapy is indicated to reduce the incidence of coronary artery abnormalities when given within the first 10 days of the illness. Aspirin is part of the therapy. Mucous membranes, conjunctiva, changes in the extremities, and cardiac involvement are seen. The fever of Kawasaki disease is unresponsive to antibiotics. It is responsive to antiinflammatory doses of aspirin and antipyretics.

A chest x-ray examination is ordered for a child with suspected cardiac problems. The child's parent asks the nurse, "What will the x-ray show about the heart?" The nurse's response should be based on knowledge that the x-ray film will do which of the following? a. Show bones of chest but not the heart b. Evaluate the vascular anatomy outside of the heart c. Show a graphic measure of electrical activity of the heart d. Provide information on heart size and pulmonary blood flow patterns

ANS: D Chest x-ray films provide information on the size of the heart and pulmonary blood flow patterns. The bones of the chest are visible on the chest x-ray film, but the heart and blood vessels are also seen. Magnetic resonance imaging is a noninvasive technique that allows for evaluation of vascular anatomy outside of the heart. A graphic measure of electrical activity of the heart is provided by electrocardiography.

The physician suggests that surgery be performed for patent ductus arteriosus (PDA) to prevent which of the following complications? a. Hypoxemia b. Right-to-left shunt of blood c. Decreased workload on left side of heart d. Pulmonary vascular congestion

ANS: D In PDA, blood flows from the higher pressure aorta into the lower pressure pulmonary vein, resulting in increased pulmonary blood flow. This creates pulmonary vascular congestion. Hypoxemia usually results from defects with mixed blood flow and decreased pulmonary blood flow. The shunt is from left to right in a PDA. The closure would stop this. There is increased workload on the left side of the heart with a PDA.

The infant with congestive heart failure (CHF) has a need for: a. decreased fat. b. increased fluids. c. decreased protein. d. increased calories.

ANS: D Infants with CHF have a greater metabolic rate because of poor cardiac function and increased heart and respiratory rates. Their caloric needs are greater than those of average infants, yet their ability to take in calories is diminished by their fatigue. The diet should include increased protein and increased fat to facilitate the child's intake of sufficient calories. Fluids must be carefully monitored because of the CHF.

A 3-month-old infant has a hypercyanotic spell. The nurse's first action should be which of the following? a. Assess for neurologic defects. b. Prepare family for imminent death. c. Begin cardiopulmonary resuscitation. d. Place child in the knee-chest position.

ANS: D The first action is to place the infant in the knee-chest position. Blow-by oxygen may be indicated. Neurologic defects are unlikely. Preparing the family for imminent death or beginning cardiopulmonary resuscitation should be unnecessary. The child is assessed for airway, breathing, and circulation. Often, calming the child and administering oxygen and morphine can alleviate the hypercyanotic spell.

An 8-year-old girl is receiving a blood transfusion when the nurse notes that she has developed precordial pain, dyspnea, distended neck veins, slight cyanosis, and a dry cough. These manifestations are most suggestive of which of the following complications? a. Air embolism b. Allergic reaction c. Hemolytic reaction d. Circulatory overload

ANS: D The signs of circulatory overload include distended neck veins, hypertension, crackles, dry cough, cyanosis, and precordial pain. Signs of air embolism are sudden difficulty breathing, sharp pain in the chest, and apprehension. Urticaria, pruritus, flushing, asthmatic wheezing, and laryngeal edema are signs and symptoms of allergic reactions. Hemolytic reactions are characterized by chills, shaking, fever, pain at infusion site, nausea, vomiting, tightness in chest, flank pain, red or black urine, and progressive signs of shock and renal failure.

The nurse is caring for a school-age girl who has had a cardiac catheterization. The child tells the nurse that her bandage is "too wet." The nurse finds the bandage and bed soaked with blood. The most appropriate initial nursing action is which of the following? a. Notify the physician. b. Place child in Trendelenburg position. c. Apply a new bandage with more pressure. d. Apply direct pressure above catheterization site.

ANS: D When bleeding occurs, direct continuous pressure is applied 2.5 cm (1 inch) above the percutaneous skin site to localize pressure on the vessel puncture. The physician can be notified and a new bandage with more pressure can be applied after pressure is applied. The nurse can have someone else notify the physician while the pressure is being maintained. Trendelenburg position would not be a helpful intervention. It would increase the drainage from the lower extremities.

What information should the nurse teach workers at a daycare center about RSV? a. RSV is transmitted through particles in the air. b. RSV can live on skin or paper for up to a few seconds after contact. c. RSV can survive on nonporous surfaces for about 60 minutes. d. Frequent handwashing can decrease the spread of the virus.

ANS: D Feedback A RSV infection is not airborne. It is acquired mainly through contact with contaminated surfaces. B RSV can live on skin or paper for up to 1 hour. C RSV can live on cribs and other nonporous surfaces for up to 6 hours. D Meticulous handwashing can decrease the spread of organisms.

Why do infants and young children quickly have respiratory distress in acute and chronic alterations of the respiratory system? a. They have a widened, shorter airway. b. There is a defect in their sucking ability. c. The gag reflex increases mucus production. d. Mucus and edema obstruct small airways

ANS: D Feedback A The airway in infants and young children is narrower, not wider. B Sucking is not necessarily related to problems with the airway. C The gag reflex is necessary to prevent aspiration. It does not produce mucus. D The airway in infants and young children is narrower, and respiratory distress can occur quickly because mucus and edema can cause obstruction to their small airways.

What is an appropriate beverage for the nurse to give to a child who had a tonsillectomy earlier in the day? a. Chocolate ice cream b. Orange juice c. Fruit punch d. Apple juice

ANS: D Feedback A The child can have full liquids on the second postoperative day. B Citrus drinks are not offered because they can irritate the throat. C Red liquids are avoided because they give the appearance of blood if vomited. D The child can have clear, cool liquids when fully awake.

A child has had cold symptoms for more than 2 weeks, a headache, nasal congestion with purulent nasal drainage, facial tenderness, and a cough that increases during sleep. The nurse recognizes that these symptoms are characteristic of which respiratory condition? a. Allergic rhinitis b. Bronchitis c. Asthma d. Sinusitis

ANS: D Feedback A The classic symptoms of allergic rhinitis are watery rhinorrhea, itchy nose, eyes, ears, and palate, and sneezing. Symptoms occur as long as the child is exposed to the allergen. B Bronchitis is characterized by a gradual onset of rhinitis and a cough that is initially nonproductive but may change to a loose cough. C The manifestations of asthma may vary, with wheezing being a classic sign. The symptoms presented in the question do not suggest asthma. D Sinusitis is characterized by signs and symptoms of a cold that do not improve after 14 days, a low-grade fever, nasal congestion and purulent nasal discharge, headache, tenderness, a feeling of fullness over the affected sinuses, halitosis, and a cough that increases when the child is lying down.

Which statement is characteristic of AOM? a. The etiology is unknown. b. Permanent hearing loss often results. c. It can be treated by intramuscular (IM) antibiotics. d. It is treated with a broad range of antibiotics.

ANS: D Feedback A The etiology of AOM may be Streptococcus pneumoniae, Haemophilus influenzae, and Moraxella catarrhalis, or a viral agent. Recent concerns about drug-resistant organisms have caused authorities to recommend judicious use of antibiotics and that antibiotics are not required for initial treatment. B Permanent hearing loss is not a frequent cause of properly treated AOM. C Intramuscular antibiotics are not necessary. Oral amoxicillin is the treatment of choice. D Historically AOM has been treated with a range of antibiotics, and it is the most common disorder treated with antibiotics in the ambulatory setting.

Which intervention is appropriate for the infant hospitalized with bronchiolitis? a. Position on the side with neck slightly flexed. b. Administer antibiotics as ordered. c. Restrict oral and parenteral fluids if tachypneic. d. Give cool, humidified oxygen.

ANS: D Feedback A The infant should be positioned with the head and chest elevated at a 30- to 40-degree angle and the neck slightly extended to maintain an open airway and decrease pressure on the diaphragm. B The etiology of bronchiolitis is viral. Antibiotics are only given if there is a secondary bacterial infection. C Tachypnea increases insensible fluid loss. If the infant is tachypneic, fluids are given parenterally to prevent dehydration. D Cool, humidified oxygen is given to relieve dyspnea, hypoxemia, and insensible fluid loss from tachypnea.

Which vitamin supplements are necessary for children with cystic fibrosis? a. Vitamin C and calcium b. Vitamin B6 and B12 c. Magnesium d. Vitamins A, D, E, and K

ANS: D Feedback A Vitamin C and calcium are not fat soluble. B B6 and B12 are not fat-soluble vitamins. C Magnesium is not a vitamin. D Fat-soluble vitamins are poorly absorbed because of deficient pancreatic enzymes in children with cystic fibrosis; therefore supplements are necessary.

The nurse is providing education related to "Safe Sleep" to the parents of a healthy newborn infant to help prevent sudden infant death syndrome (SIDS). The nurse instructs the parents that bed sharing is not recommended; however, they should put the infant in a safe bassinet or crib in the parent's room for sleeping. Is this statement true or false?

ANS: T The American Academy of Pediatrics (AAP) recommends the following actions to help prevent SIDS in infants: place healthy infants on their backs to sleep, use mattresses with a firm sleeping surface, avoid exposing the infant to secondhand smoke, and offer a pacifier for sleep. In addition, bed sharing is not recommended, and parents are advised to put the infant in a safe bassinet or crib in the parent's room for sleeping.

which leukemia has chrome problems

Acute lymphocytic (ALL)

EDU for anemia

Add iron rich foods to diet and folic acid Organ meats, shellfish, poultry, legumes, molasses, fortified cereals Legumes, liver, dark leafy green, lean beef, potatoes

What immunizations do you give Hodgkin's and Non-Hodgkin's Lymphoma its

Administer pneumococcal and meningococcal immunizations

Diagnosis for Leukemia

Anemia with weakness, fatigue and pallor Cough, wheezing, tracheal/bronchial compression, respiratory distress Neutropenia with infection Thrombocytopenia Pain

assessment for leukemia

Anemia with weakness, fatigue and pallor Cough, wheezing, tracheal/bronchial compression, respiratory distress Neutropenia with infection Thrombocytopenia Pain

Which of the following is most likely associated with a cerebrovascular accident (CVA) resulting from congenital heart disease? a. Polycythemia b. Cardiomyopathy c. Endocarditis d. Low blood pressure

Answer: A The child with congenital heart disease develops polycythemia resulting from an inadequate mechanism to compensate for decreased oxygen saturation.

The mother of a child with tetralogy of Fallot asks the nurse why her child has clubbed fingers. The nurse bases the response on the understanding that clubbing is due to which of the following? a) Anemia. b) Peripheral hypoxia. c) Delayed physical growth. d) Destruction of bone marrow.

Answer: B Clubbing of the fingers is one common finding in the child with persistent hypoxia leading to tissue changes in the body because of the low oxygen content of the blood (hypoxemia). It apparently results from tissue fibrosis and hypertrophy from the hypoxemia and from an increase in capillaries in the area, which occur as the body attempts to improve blood supply. Clubbing of the fingers is associated with polycythemia, not anemia. Polycythemia results from the body's attempt to increase oxygen levels in the tissues. The child may be small for his or her chronological age, but clubbing does not result from slow physical growth. Destruction of the bone marrow is not related to this congenital heart malformation. Instead, bone marrow is actively producing erythrocytes to compensate for the chronic hypoxia.

A nurse is preparing for the admission of a child with a diagnosis of acute-stage Kawasaki disease. On assessment of the child, the nurse expects to note which clinical manifestation of the acute stage of the disease? a) cracked lips b) a normal appearance c) conjunctival hyperemia d) desquamation of the skin

Answer: C In the acute stage, the child has a fever, conjunctival hyperemia, red throat, swollen hands, rash, and enlargement of the cervical lymph nodes. In the subacute stage, cracking lips and fissures, desquamation of the skin on the tips of the fingers and toes, joint pain, cardiac manifestations, and thrombocytosis occur. In the convalescent stage, the child appears normal, but signs of inflammation may be present.

A 16 month old child diagnosed with Kawasaki Disease (KD) is very irritable,refuses to eat, and exhibits peeling skin on the hands and feet. Which of the following would the nurse interpret as the priority? a) applying lotions to the hands and feet b) offering foods the toddler likes c) placing the toddler in a quiet environment d) encouraging the parents to get some rest

Answer: C One of the characteristics of children with KD is irritability. They are often inconsolable.Placing the child in a quiet environment may help quiet the child and reduce the workload of the heart. The child's irritability takes priority over peeling of the skin.

A child diagnosed with tetralogy of fallot becomes upset, crying and thrashing around when a blood specimen is obtained. The child's color becomes blue and respiratory rate increases to 44 bpm. Which of the following actions would the nurse do first? a) obtain an order for sedation for the child b) assess for an irregular heart rate and rhythm c) explain to the child that it will only hurt for a short time d) place the child in knee-to-chest position

Answer: D. the child is experiencing a "tet spell" or hypoxic episode. Therefore the nurse should place the child in a knee-to-chest position. Flexing the legs reduces venous flow of blood from lower extremities and reduces the volume of blood being shunted through the interventricular septal defect and the overriding aorta in the child with tetralogy of fallot. As a result, the blood then entering the systemic circulation has higher oxygen content, and dyspnea is reduced. Flexing the legs also increases vascular resistance and pressure in the left ventricle. An infant often assumes a knee-to-chest position to relieve dyspnea. If this position is ineffective, then the child may need sedative. Once the child is in this position, the nurse may assess for an irregular heart rate and rhythm. Explaining tho the child that it will only hurt for a short time does nothing to alleviate hypoxia.

What kind of trait is sickle cell disease

Autosomal recessive trait

A nurse is assessing a child who has leukemia. Whicho f the following are early manifestations of leukemia? SELECT ALL THAT APPLY A. Hematuria B. Anorexia C. Petechiae D. Ulcerations in the mouth E. Unsteady gait

B Anorexia C Petechiae E unsteady gait

How do you position a child after cleft lip repair?

Back or side-lying position Want to protect suture line

When is the best time for a cleft palate repair to be done?

Before speech develops, usually between ages 1 and 2 years

Hodgkin's and Non-Hodgkin's Lymphoma Interventions

Chemotherapy Administer pneumococcal and meningococcal immunizations Prevent infection using standard precautions and isolation as required

clinical manifestation for Hodgkins and Non-hodgkins lymphoma

Common enlarged cervical or supraclavicular lymphadenopathy Fever, weight loss, night sweats , cough, abdominal discomfort , anorexia, nausea, pruritus

The 3 C's

Coryza, cough and conjunctivitis ; found in rubeola (measles)

Patho for aplastic anemia

Deficiency of the formation of blood elements o Pancytopenia is decreased leukocytes, platelets, and erythrocytes

What restraint do you use after cleft lip repair?

Elbow restraints

What other position can we place GER patients in?

Elevated prone position (30 degrees) to decrease reflux and improve stomach emptying to prevent aspiration

Iron Deficiency Anemia risk fACTORS

Excessive cow's milk intake in toddlers*** KNOW Malabsorption disorders Poor dietary intake Increased iron requirements (blood loss)

Aplastic Anemia:

Extreme anemia as a result of decreased RBC production

women and hemophilia

Females are carriers and do not have disease

How often do you burp infants with cleft lip/palate?

Frequently b/c swallow lots of air

What is the difference between gastroesophageal reflux and gastroesophageal reflux disease?

GER is the passage of gastric contents into the esophagus GERD is the chronic form of this

What is celiac disease?

Genetic malabsorption disorder where there is *permanent* intestinal intolerance to gluten

The childhood vaccine ____________________ has dramatically reduced the incidence of epiglottitis.

H. influenzae type B (HIB) vaccine The nurse should encourage parents of young children to have their children immunized against H. influenzae to decrease the risk for contracting epiglottitis. Prophylaxis with rifampin is given to underimmunized contacts or family members younger than 4 years old and to any child contact who is immune depressed.

Immunizations for SCD to prevent again

HIV pneumococcal and meningococcal organisms

IDP for thrombocytopenia

Idiopathic

when solids are introduce

Iron fortified cerea

Do you correct cleft lip or palate first?

Lip Wait to fix palate until bone forms b/c mainly cartilage at birth

Iron administration

Liquid dose sipped through straw because can color teeth Use Z track method for iron dextran (Imferon)

How do you feed cleft lip/palate infants?

Long nipples or syringe to side of mouth

earl manifestation for leukemia

Low-grade fever, pallor, increased bruising and petechiae, listlessness, enlarged liver, lymph nodes, and joints, abdominal, leg and joint pain, constipation, HA, vomiting and anorexia, unsteady gait

Do we put things in their mouth after cleft palate repair?

No Soft diet until well healed

What is the main concern for a cleft lip/palate?

Nutrition

Vaso Occlusive crisis

Obstruction of blood flow causing tissue hypoxia and necrosis

Late manifestation for leukemia

Pain, hematuria, ulcerations In mouth, enlarged kidneys and testicles, increased ICP

What is intussusception?

Piece of bowel goes backward inside itself, forming an obstruction

What structures are damaged with GERD?

Resp structures Leads to pneumonia and bronchospasms

Iron Deficiency Anemia manifestations

Signs and Symptoms: Poor muscle tone Prone to infection Skin may be described as porcelainlike ( fair light skin) Edematous Retarded growth- growth can be stunted Decreased serum concentration of the proteins albumin and gamma globulin Tachycardia Pallor Brittle spoon-shaped nails Fatigue and irritability

How do we alter feedings to help reflux?

Small frequent feedings and thickened formula

How do we diagnose pyloric stenosis?

Ultrasound

How do we position patients with GER with feedings? At night?

Upright with feedings and at night

Nursing interventions for anemia

Use commercial iron-fortified formula Preterm = iron by 2 months → full term = iron by 4-6 months High iron and vitamin C diet Iron fortified cereal when solids are introduced Expect tarry green stools Don't want to give with dairy because it will bind to calcium - give 1 hour before or 2 hours after milk consumption o Liquid dosage of iron sipped through a plastic straw to avoid colored tooth enamel, brush teeth after

Painful episode with hand-foot syndrome causing symmetrical infarct in the bones of the hands and feet and very painful swelling of soft tissue

Vaso Occlusive crisis

How do you clean suture lines for cleft lip?

With normal saline

Are speech defects common after cleft palate repair?

Yes

Hemophilia A and B are

are inherited X-linked recessive traits that affect only males

what will raise in Vaso Occlusive crisis

bilibruin and reticulocyte

def of hemophilia

deficiency of factor VIII, which is produced by the liver and assists with thromboplastin formation in blood coagulation

Thrombocytopenia causative agents (2)

drugs infection

When is Hemophilia diagnosed?

first at curcumicsion

what diet for anemic

high iron and fit c

As a child with asthma struggles to get enough air, the respiratory rate increases (tachypnea). Tachypnea lowers the carbon dioxide levels in the blood. This is known as _____________.

hypocapnia As the child tires from the increased work of breathing, hyperventilation occurs and carbon dioxide levels increase. Increased levels of carbon dioxide in the blood (hypercapnia) during an asthma episode may be a sign of severe airway obstruction and impending respiratory failure.

when should sickle cell pt recieve medial care when it comes to fever

if temp is over 38.3 c or 101 F

Anemia results form

inadequate dietary iron

Hodgkins disease involves

involves the lymph nodes with Reed-Sternberg cells

preterm

iron by 2 mo

full term

iron by 4-6 mo

what kind of formal for enima

iron fortified formula

most common cancer of childhood

leukemia

increase risk of getting leukemia

male caucation between 2-5 years old fam hx

4 types of leukemia

o Acute lymphocytic (ALL) (chromsome problems) o Acute myeloblastic (AML) → more severe o Chronic myeloblastic (CML) o Chronic Lymphocytic (CLL)

PT edu for hemophilia

o Avoid aspirin it interferes with platelet function à Tylenol instead o Avoid heat or ice o Exercise and physical therapy - supportive care o Provide soft toothbrushes and venipuncture instead of finger sticks to decrease bleeding o Medical alert tag o Administration of factor VIII o Administration of DDAVP (1-deamino-8D-arginine vasopressin) Help with platelet aggregation o Teach parents to prevent bleeding at home " Safe environment, safe play " Low contact sports (bowling, golfing, fishing, swimming) allowed " Soft-bristled toothbrush o Encourage reg. exercise and physical therapy when bleeding in controlled o Teach control of bleeding episodes (RICE)

Patho for sickle cell disease

partial or complete replacement of abnormal hemoglobin S for normal hemoglobin A

Why are adolescents at risk for anemia d/t ___ ___ __ __

poor diet rapid growth menses strenuous activities obesity

3 things to tell parents to do when child has sickle cell disease

proper hydration infection prevention immunization

What do Sickle cell pt get before dental procedure

prophylactic penicillin

5 things you will see with thrombocytopenia

rocky mountain spotted fever colorado tick fever malaise bacteria infection HIT

What happened to deformed cells in sickle cell disease

round to sickle ( crescent) shape

How to take liquid iron sipped through a plastic straw to avoid colored tooth enamel, brush teeth after

sipped through a plastic straw to avoid colored tooth enamel, brush teeth after

When both parents have the trait for sickle cell disease

there is a 1 in 4 chance of each press that child will have SCD

Cause of thrombocytopenia

unknown


Set pelajaran terkait

220-901 A+ Command Line Tools & Troubleshooting

View Set

Lesson 6: Annuities and Retirement Plans

View Set

ADP, ATP, and Cellular Respiration.

View Set

XCEL Chapter 5: Life insurance underwriting and policy issue

View Set

EXAMFX Health Insurance: Field Underwriting Procedures

View Set

Chapter 17: Nations and Empires, 1850-1914 Quiz

View Set